You are on page 1of 75

Situation

As a new public health nurses at a Rural Health Unit, Ms. Casey reviews basic concepts related to her position
and job.
1. Ms. Casey know that the PRIMARY goal of community health nursing is to ________.
a. Enhance the capacity of individuals, families and communities manage their health needs
b. Contribute to people’s well-being through sustainable development goals projects
c. Increase the productivity of the people by providing them with health services
d. Support and supplement the efforts of the medical profession in illness preventions
2. Community health nursing (CHN) is a field of nursing practice. Which best explains this statement?
a. The scope of practice depends on the health needs and problems identified.
b. The services are delivered outside of purely curative institutions.
c. The service are provided along with community health volunteers.
d. The practice is conducted in the geographical location of people.
3. Ms. Casey is aware that she will be dealing with population as a client. Which of the following provides the
BEST definition of a population?
a. Cluster of individuals who are at-risk of certain health conditions.
b. Group of people sharing the same geographic environment.
c. Collection of people who are beneficiaries of health services.
d. Collection of individuals who share common characteristics.
4. An example of an aggregate or population that Ms. Casey should consider in program planning would
consist of ______.
a. Patients confined in a district hospital
b. Christians in the community
c. Young children and adolescents attending school
d. People playing games three times a week in a senior citizen club
5. Which Nursing activity is unique to community/public health nurses like Ms. Casey?
a. Care for sick and dying
b. Promote livelihood projects assist poor families
c. Home visits to family clients referred by the health center physician
d. Focus on vulnerable groups in the community

Situation
You are making a home visit to Mr. and Mrs. Baker who are both in their seventies. Communicating with older
persons requires effective and therapeutic techniques.
6. You noted that Mrs. Baker frequently shifts the conversation to reminisce. Which of the following
communication techniques would be most effective with Mrs. Baker?
a. Asking exploring questions
b. Changing the subject
c. Giving information
d. Restarting
7. The assessment of the older adult should focus on:
a. Functional level
b. Social Functioning
c. Relationship needs
d. Chronological age
8. When communicating with the cognitively elderly clients, it is important to understand the use of touch.
Which of the following is true in relation to using of touch? Which of the following is true in relation of using
touch with the elderly?
a. It is calming.
b. The use of touch must be determined on an individual basis.
c. It increases their agitation and confusion
d. The face should be touched to establish rapport.
9. Mr. and Mrs. Baker are an elderly couple that you are interviewing at home. Mr. Baker asks to speak you
alone. He tells you that Mrs. Baker has become forgetful lately and yesterday he received an overdue water
bill. He asks, “Should I take over this responsibility?” What would you say to Mr. Baker?
a. “Old people get forgetful at times”
b. “Could you tell Mrs. Baker that you will help with the bill paying?”
c. “Tell Mrs. Joson that you will assume the responsibility now.”
d. “You don’t know what to suggest. This is a family matter.”

10. It is critical to assess hearing loss in the elderly because:


a. Only adults can only hear at 90 words per minute
b. There is an initial inability to hear low frequency consonants
c. Deafness causes hair loss
d. Hearing loss has a direct impact on communication

Situation
Nurse Giselle makes home visits to assigned barangays and meets families of different types.
11. One such family is a married couple who has two biologic children living with them as well as a child from
the wife’s first marriage. What type of family is evident?
a. Homogenous
b. Extended
c. Blended
d. Nuclear
12.Nurse Giselle encounters a married couple who is raising three children. Recently, the wife’s mother moved
in. This family should be assessed as ___.
a. Nuclear
b. Extended
c. Alternative
d. Blended
13. When Nurse Giselle assesses a family, which family task would she consider having HIGHEST PRIORITY for
health family functioning?
a. Reproduction of new family members
b. Physiologic maintenance and safety
c. Allocation of family resources
d. Maintenance of order and authority
14. Which documentation of family assessment indicates a healthy and functional family?
a. Member provide loving and mutual support
b. Under stress, members turn inward so as not to be burden others.
c. Members believe they can depend in each other.
d. Husband holds dominant power over his wife
15. Which of the following scenario BEST demonstrates a continuing healthy family?
a. A couple renews their marital relationship after their children become adults
b. A couple requires their adolescent children to attend church services three times a week
c. One parent takes care of children. The other parent earns income and maintains the home
d. A family has strict boundaries that require members to address problems within the family.

Situation
Nurse Che is a part of health team of a Rural Health Unit that implements the Philippine Package of Essential
NCD Intervention (PHILPEN). He is particularly assigned to conduct “hypertension screening” among 15 years
old and above during home visits.
16. The MOST accurate and reliable technique for indirect BP measurement is the _______.
a. auscultatory using a mercury manometer.
b. palpation using mercury manometer
c. auscultatory using an aneroid BP apparatus
d. palpation using an aneroid BP apparatus
17. Nurse Che remembers that blood pressure taken in the upper arm versus the forearm varies. Thus, there is
a need to document which site is used. Which of the following values is the correct variation in mmHg?
a. 10-15
b. 5-10
c. 14-20
d. 1-4
18. Nurse Che needs to expose the patient’s arm and position it with palm upward and the arm slightly flexed
with the whole arm supported at which level?
a. Above heart
b. Below heart
c.Heart level
d.Any level
19. Nurse Che took the blood pressure of a 45 year old family member and got the reading as 146/92 mmHg.
This reading is classified as:
a. Hypertension, stage 1
b. Hypertension, stage
c. Hypertension, stage 2
d. Prehypertension
20. In defining and classifying hypertension, blood pressure readings must be based on measurements done
during ________.
a. at least two visits taken one week apart
b. three consecutive daily visits
c. two readings taken two weeks apart
d. two consecutive monthly visits

Situation
Public health nurses like Ms. Jam should be knowledgeable on the importance of immunity in the prevention
of communicable diseases by building the capacity of the body to restrict harmful microorganisms or viruses
from entering it.
21. Nurse Jam knows that the type of immunity which is longest acting is____________
a. Natural immunity
b. Passive immunity
c. Artificial immunity
d. Active immunity
22. She learned that the MOST important function of inflammation and immunity is___________
a. Preventing any entry of foreign material
b. Providing protection against invading organisms
c. Regulating the process of self- tolerance
d. Destroying bacteria before damage occurs
23. Nurse Jam administers hepatitis B immunoglobin serum to Helen, which will provide her with passive
immunity. One advantage of passive immunity is that it_____
a. Encourages the body to produce antibodies
b. Has effects that last a long time
c. Is highly effective in the treatment of disease
d. Offers immediate protection
24. Nurse Jam explains to a mother whose ten year child just received a tetanus- toxoid injection that the
toxoid vaccine confers which of the following immunity?
a. Long lasting active immunity
b. Lifelong natural immunity
c. Intermediate artificial immunity
d. Short-acting passive immunity
25. Nurse Jam knows that one of the following is NOT a vaccine preventable disease, hence no immunity can
be given to a child.
a. Measle
b. Polio
c. Hepatitis B
d. Asthma

Situation
A strong typhoon hits a rural community in the Visayas where 300 families are left without food and shelter.
26. Which reaction demonstrates the community’s readiness?
a. Community leaders activate the community disaster plan.
b. Community leaders contact the National Disaster and Relief Center about the disaster.
c. Residents run to DSWD local office to ask for donation of food and water
d. Typhoon victims are relocated to church patios, school gyms and municipal playground.
27. Nurse Charlie is working in a temporary shelter of victims immediately following a natural disaster. Which
condition is the nurse most likely to encounter?
a. Hallucination
b. Stress
c. Communicable disease
d. Chronic illness
28. Which task would a nurse be most likely to perform while volunteering on a disaster medical assistant
team?
a. Assist in triaging disaster victims.
b. Conduct a needs disaster assessment of the community
c. Set up immunization clinics to administer anti tetanus vaccine
d. Provide medical care for disaster victims
29. Which population has a GREATER risk for disruption after disaster?
1) Single-parent families
2) Children
3) Middle income families
4) Substance abusers
5) Health care workers
a. 1, 2, and 4
b. 2, 3, and 4
c. 1, 3, and 5
d. 2, 4 and 5
30. After the devastating typhoon hits the community, a grassroots organization is formed to improve
conditions post recovery. Nurse Charlie joins the organization and volunteers to contact and meet with public
officials and key informal leaders about the community’s concerns. What role does Nurse Charlie
demonstrate?
a. Educator
b. Advocate
c. Citizen
d. Political ally

Situation
Promoting metal is equally important in promoting physical health. Unfortunately, metal health has been
largely neglected.
31. Public Health Nurse Sarah is concerned about the impact of metal health on the community. She is
preparing for a presentation about mental health in the Philippines. Which of the following information should
be included?
a. Mental illness is an increasing problem among the upper class.
b. The incidence of depression is expected to decrease within the next 10 months.
c. Almost one per 100 household has a member with mental disability.
d. Post traumatic stress disorder is the most common of anxiety disorders.
32. The PRIMARY reason that mental illness often results in worsening of physical health problems is because
of the _______.
a. Loss of cognitive function
b. Feelings of inadequacy
c. Inability or lack of motivation for effective self-care
d. Side effects of mental health modification
33. Nurse Sarah shared the statistics that intentional self-harm was found to be the 9th leading cause of death
among 20-24 years old Filipino adults. Increasing number of mental health problems may be related to
_______. (Select all that apply)
1. Poverty
2. Age of the parents
3. Family unemployment
4. Changes in family structure
5. Lost of community social support
a. 1, 2, and 5
b. 1, 3, 4, and 5
c. 1, 2, 3 and 4
d. 1, 2 and 3
34. Nurse Sarah noted that there were number of poor older women, living alone, who are suffering from
depression. Her FIRST goal is to ______.
a. Improved quality of life
b. Provide recreation and entertainment
c. Reduce suicide risk
d. Improve level of function
35. In promoting mental health, Nurse Sarah is cognizant of her nursing responsibilities. These include the
following, EXCEPT:
a. Teach parents the importance of community of providing emotional support to their children.
b. Increase general knowledge of the community on knowledge on mental hygiene.
c. Help the community address factors that promote mental well-being.
d. Refer at once cases of bizarre behaviors to a psychiatrist.

Situation
The World Health Organization (WHO) stated that “climate change is the biggest threat to public health this
century.” Climate change is happening, and scientists believe that global warming results from human activity.
36. As a public health nurse and as a citizen, you are concerned about the Earth’s warming up fast from the
production of greenhouse gases which are involved in the greenhouse effect. The most common greenhouses
gases are the following, EXCEPT:
a. Oxygen
b. Methane
c. Water Vapor
d. Carbon dioxide
37. Along with the natural greenhouse gases, there are some man-made ones that keep on increasing in the
atmosphere that contribute to the heating up of the Earth’s surface. These HARMFUL human activities include
the following:
1) use of liquefied petroleum gases
2) burning of fossil fuels
3) use of aerosols and plastics
4) cutting down trees
5) burning waste materials
6) use of recycled paper
a. 1, 2, 5, and 6
b. 2, 4, 5 and 6
c. 2, 3, 4, and 5
d. 1, 3, 4, and 6
38. You are aware of the consequences of the Earth’s climate getting warm on the weather which eventually
affect people. Which of the following is NOT included?
a. storms and floods
b. cool summer
c. droughts
d. more rains and winters
39. You are concerned about the effects of global warming on the health of people. In your advocacy, which of
the following statements should NOT be included?
a. Increased aeroallergens result in increase in cardiovascular disease.
b. There is an increase in mental health problems due to harmful and unexpected consequences.
c. Increased exposure to ultraviolet rays results in increases in skin cancers.
d. Changing patterns result in increases in vector, food and waterborne disease
40. The following messages can be included in your advocacy helping reduce the generation of greenhouse
gases and slow down global warming, EXCEPT:
a. Recycle used materials
b. Plant more trees
c. Use electricity for cooking
d. Walk or use bicycle
41. The public health nurse is aware of the importance of the need to perform roles in preparing for a disaster.
The BEST example of one of these roles would be to:
a. Assist in the prevention of injury
b. Identify disaster risks
c. Coordinate emergency care
d. Inspect cluster of houses
42. The residents and local leaders in Municipality X is mapping out potential disaster locations in the
community. This period of planning and preparation is described as:
a. Recovery stage
b. Pre- disaster stage
c. Impact stage
d. Non- disaster stage
43. The nurse provides counselling for victims of flood. This is an example of _____.
a. Primary Prevention
b. Secondary Prevention
c. Mental health care
d. Tertiary Prevention
44. Should a terrorist attack occur, nurses need to be ready, not only in providing quality care for the victim
but ensuring their own safety as well. During a mass disaster drill stimulating a terrorist attack, the nurse must
triage numerous severely ill persons. The client who should receive PRIORITY care is:
a. Cyanotic and not breathing
b. Apneic and has an apical rate of 50
c. Having seizures and urine incontinence
d. Gasping for breathing and are conscious
45. The nurse is considering how best to educate the community about the potential threats to health from
terrorism. What are the BEST ways to accomplish this?
1. Hold public forum to educate the community about coping with psychology effects.
2. Raise community awareness about increased immigrants to the population
3. Teach the community about sealing windows and doors in the event of a chemical attack.
4. Raise community awareness about the signs and symptoms of potential biologic agents.
a. 2 and 4
b. 1 and 3
c. 2 and 3
d. 1 and 4

Situation
According to the World Health Organization (WHO), there are major causes of non- communicable disease
that pose challenges to the live and health of millions of people and threaten economic and social
development of countries.
46. Which of the following are the four chronic disease referred to by WHO?
1. Cardiovascular Disease
2. Cancer
3. Dementia
4. Arthritis
5. Diabetes Mellitus
6. Chronic Obstructive Lung Disease
a. 1,2,3 and 4
b. 1,2,5 and 6
c. 2,3,4 and 5
d. 2,4,5 and 6
47. The WHO reveals that the NUMBER ONE cause of death from chronic diseases worldwide, using the 2015
estimate, is _______.
a. Tuberculosis
b. Diabetes Mellitus
c. Ischemic Heart Disease
d. Pneumonia
48. In the Philippines, which is the TOP Killer according to the Department of Health (2009 data)?
a. Asthma
b. Cancer
c. Tuberculosis
d. Cardiovascular Diseases
49. When studying chronic diseases, the multifactorial etiology of illness is considered. What does this imply?
a. Single organism that causes the disease, such as cholera, must be studied in more detail.
b. Focus should be on the factors or combinations and levels of factors contributing to disease.
c. The rise in infectious and communicable disease must be the main focus.
d. Genetics and molecular structure of disease is paramount.
50. Determinants of health to address the development of cancer in a community include:
1. Proximity of the community to chemical plants that emit poisonous gases.
2. High percentage of tobacco use among the residents.
3. Prevailing diet high in processed food and fat.
4. Availability of the health facilities.
5. Men age of women population.
a. 3,4 and 5
b. 1,2 and 3
c. 1,3 and 4
d. 2,4 and 5

Situation
Interdisciplinary teamwork is an important model for delivering health care to patients. Integrating services
among many health providers leads to better outcomes delivered to underserved populations and
communities with limited access to health care.
51. Professional group productivity and member satisfaction are best achieved by which of the following
leadership styles?
a. Democratic
b. Authoritarian
c. Laissez-faire
d. Autocratic
52. As a member of a professional work-related group you know that:
a. Functional goals are assigned by management
b. The leader is responsible for overall functioning
c. The members summarize progress and allocate assignments
d. The cooperation of all members is required.
53. The nurse is making a referral for the client. What should be the first consideration for the nurse to do?
a. Acceptability of the referral to the client
b. Cost of the referral service
c. Barriers to making a referral
d. Client’s eligibility for the service
54. Group with a laissez faire leader are likely to be ___.
a. More structured in format
b. Less productive
c. More productive
d. More satisfying to members
55. You have volunteered to be part of a work group whose purpose is to look at ways to prevent medication
error. You know that:
a. Teamwork enhances the probability of goal achievement
b. Group process is not important in a task
c. The leader assumes responsibility for the overall group functioning
d. The goal should be set by management
Situation
Nurse Gabriella has just passed the Nursing Licensure Examination (NLE). She directly needs to refresh herself
regarding the law that governs the practice of nursing the Philippines.
56. By virtue of Section 16 of the Philippine Nursing Law, the first step Nurse Gabriella needs to do after
passing the NLE is to:
a. Apply for the Certificate of Registration/Professional License.
b. Get an official copy of her board rating from the PRC office
c. Apply for a Professional Identification Card.
d. Take oath of profession before the Philippine Regulatory Board of Nursing
57. Which government body has the power to revoke or suspend Nurse Gabriella’s certificate of
registration/professional license, should she commit unprofessional and unethical conduct in the future?
a. Philippine Nurses Association
b. Professional Regulation Commission
c. Commission on Higher Education
d. Professional Regulatory Board of Nursing
58. What Republic Act provides for a more responsive nursing profession?
a. RA 9173
b. RA 7164
c. RA 9371
d. RA 7614
59. Nurse Gabriella intends to work as a Public Health Nurse. Which part of the Nursing Law should Nurse
Gabriella refer to for her safe practice as a novice nurse?
a. Nursing Research
b. Nursing Personnel System
c. Scope of Nursing Practice
d. Nursing Education
60. When Nurse Gabriella eventually gets employed in the Rural Health Unit, foremost, she is required to
utilize which of the following in rendering care to her clients in the community?
a. Nursing Process
b. Therapeutic use of self
c. Health teachings
d. Traditional and innovative approaches

Situation
Public Health Nurse Mandy Reyes asks the new nurses about their membership to the accredited professional
nursing organization and their plans for personal and professional growth.
61. Which of the following statement is true about membership to the Philippine Nurses Association (PNA)? It
is:
a. Voluntary and is encourage by the PRC code of ethics
b. Compulsory for new nurses wanting to practice nursing in the Philippines
c. Mandatory as stipulated in the Philippine Nursing Act if 2002
d. Required for those employed in government health facilities
62. Why would a nurse choose to be an active member of several professional associations? PRIMARILY to:
a. Participate in the discussion and political action activities concerning the profession
b. Able to spend time with friends and colleagues
c. Part of the organization’s annual convention/conference.
d. Be able to have professional development opportunities
63. Which of the following is the MOST important step to developing cultural competence for the nurse?
a. understand and recognize her own cultural background
b. understand the culture of her client
c. identify the goals of cultural-congruent goals
d. gain proficiency in another language and dialect
64. Quality of client care is ensured not only by continuing education. Professional responsibility along this line
requires, FOREMOST:
a. knowing the standards of practice
b. understand the socioeconomic background of patients
c. getting a master’s degree in nursing
d. a nurse-centered care
65. Quality of client care is ensured not only by continuing education. Professional responsibility along this line
requires, FOREMOST:
A. Obtaining an advance academic degree in nursing
B. Attending as many as nursing conference's
C. Conforming to institution’s policies and guidelines
D. Monitoring one’s own practice according to standard set by the profession

Situation
As a nurse, knowledge of the legal aspects in nursing is essential to safeguard self and clients from legal
complications. Clients are becoming increasingly aware of their health rights, thus you should recognize your
professional boundaries and consequences of nonconformity.
66. Your co-worker got a thank you card from a patient’s family with an accompanying photo of her with the
family. She asked you if it would be alright to post the post on facebook. Which would be your BEST response?
a. “ I think that would be okay, but you should ask permission from our head.”
b. “ Yes, as long as you ask the family if that would be okay.”
c. “No, that could lead to a malpratice suit by the patient or family.”
d. “No, posting pictures of patients and families on social media is not acceptable.”
67. Identify which action is acceptable as an exception to a nurse’s obligation regarding confidentiality.
a. Leaving printouts of lab reports on the desk in the physicians’ lounge
b. Discussing a patient’s care with someone who is not involved in patient care
c. Discussing a patient’s condition in a public place as long as his name is not mentioned
d. Reporting certain disease to public health authority
68. The central question in any charge of malpractice is whether the prevailing standard of care was met.
Which of the following statements are TRUE about standards of nursing care?
1. Basic prudent nursing care is a standard.
2. Any health care provider can determine standards of care.
3. Standards of care are never changing.
4. Standards are based on the ethical principle of non- maleficence.
5. National standards of nursing practice are standards for all nurses.
a. 1,3,4
b. 2,3,5
c. 1,4,5
d. 2,4,5
69. Select the nursing responsibility which can NEVER be delegated.
a. Rotation schedule
b. Evaluation
c. Accountability
d. Complex Procedure
70. For safe nursing practice, you should observe some of the do’s and do not’s in performing your
professional duty. Which ones are NOT acceptable standard of actions by the nurse?
1. Keeping current year license to practice
2. Being a witness to a patient’s will
3. Following policies and procedures
4. Protecting patients from injuring themselves
5. Accepting money or gift from patients
6. Giving practical advice contrary to the doctor’s orders
a. 1,4, 6
b. 1,3,4
c. 2,4,5
d. 5,6

Situation
The Code of Ethics of Registered Nurses in the Philippines was promulgated by the Professional Regulatory
Board of Nursing (PRBON) in consultation with national nursing organizations.
71. The Code’s Preamble recognizes that the PRIMARY responsibility of the nurse is to _______.
a. treat and cure simple disease
b. preserve health at all cost
c. support client in health matters
d. achieve quality of life for clients
72. Article II of the Code outline the ethical principles governing the Registered Nurses and the People. In
relation to this, the following are guidelines the nurse must observe, EXCEPT:
a. In the event of conflicts, culture and values take precedence over welfare and safety.
b. Respect the spiritual beliefs and practices regarding diet and treatment.
c. consider the individuality and totality of patients when giving care
d. Uphold the dignity and rights of individuals.
73. As a citizen and a member of society, the nurse must:
1) Be involved in community concerns and projects
2) Be equipped with knowledge of community health resources.
3) Lead their lives in conformity with the principles of right conduct.
4) Project an image that will uplift the nursing profession at all times.
5) Be on guard that her/his words and actions are always agreeable with others
A. 2, 3, and 5 only
B. 2, 3, 4, and 5
C. 1, 2, 3 only
D. 1, 2, 3, and 4
74. As a professional, the nurse must always conduct herself appropriately. Which of the following behaviors
does NOT correspond to this statement?
a. Avoid any abuse of the privilege relationship which exists with patients.
b. Accept gift or hospitality from any person in exchange for a requested favor.
c. Not demand and receive any commission, fee or emolument for referrals made.
d. Not allowing themselves to be used in advertisement that would demean the image of profession.
75. In the said Code of Ethics for Nurses, the nurse in relation to her profession, is expected to do the
following, EXCEPT:
a. Participate actively in the growth and development of the nursing profession
b. Strictly adhere to the nursing standard set by the Professional Regulatory Board of Nursing
c. Strive to secure equitable socio-economic and work conditions for nurses.
d. Be a member of the accredited professional organization which is the International Council of Nurses.

Situation
Nurse Eli knows that ethics is important in community health nursing practice and that ethical decisions are
conducted in a universal and standard framework.
76. One step in ethical decision- making is to place an ethical issue or dilemma within a meaningful context.
The rationale for this step is:
a. The nature of ethical issues and dilemmas determines the specific ethical approach used.
b. People cannot make sound ethical decision if they cannot identify ethical issues and dilemmas
c. Multiple factors affect the interpretation and justification of the ethical issues and dilemmas
d. Health professionals cannot avoid choice and action in applying ethics in their work setting.
77. The historical character in nursing whose work in environmental health, providing care to destitute people,
and disease prevention establishing the nurse as a leader and community-oriented was _______.
a. Lilian Wald
b. Virginia Henderson
c. Margaret Sanger
d. Florence Nightingale
78. Ethical decision making is the component of ethics that focuses on ____.
a. The result of the decision
b. The process of how decision are made
c. The factors impinging on making the decision
d. Who makes the decision
79. The ethical principle that empowers the client (groups or communities) to make knowledgeable decision is
a. Act in accordance to client’s wishes
b. Act in client’s best interest
c. Offer franks and independent advice
d. Keep the client properly informed
80. When Nurse Eli cannot meet a need of the client, it is most appropriate for her to:
a. Refer the client to an agency that may be able to give assistance
b. Suggest that the client discuss the need with the physician.
c. Encourage the client to pay someone to meet that need.
d. Inform the client that insurance will not pay for the care.

Situation
It is important that every nurse knows about PhilHealth is one of the pillars in attaining the universal health
coverage.
81. Which of the following statements is NOT true about PHILHEALTH?
a. It provides health care, with the arrangement “consult now, pay later scheme.”
b. It is a means for those who can afford medical care to subsidize those who cannot.
c. It is a health insurance program for all Filipinos.
d. Beneficiaries have access to a comprehensive package of services.
82. Under PhilHealth, beneficiaries have access to a comprehensive package of services. Outpatient treatment
is given for which of the following diseases?
a. Heart disease
b. Tuberculosis
c. Cancer
d. Diabetes
83. Which of the following INPATIENT care are included in PhilHealth?
1) Room and board
2) Medicines, drugs
3) Laboratory and diagnostics
4) Professional fees
a. 4 and 5
b. 1, 2, 3, and 4
c. 1, 3, and 5
d. 1, 3, and 4
84. The case rate amount depends on the final diagnosis, each diagnosis having corresponding fixed amount
or package. The case rate amount means that:
a. Patient/Family shall pay the total bill and shall be reimbursed later by the HCI.
b. The amount that can be paid by the patient on installment basis
c. Amount shall be deducted by the Health Care Insurance (HCI) from the member’s total bill.
d. Prior to discharge, amount shall be paid in full except doctors’ fees.
85. Which of the following outpatient benefits is NOT included?
a. Contract operations
b. Cosmetic surgery
c. Animal bite treatment
d. Blood transfusion
Situation
Public health nurses serve a unique and vital role environmental health. They conduct specific nursing actions
that may promote health reducing environmental risks to health of the community.
86. Nurse Maribel proposes fluoridation of municipal drinking water of the community. In doing so, she is
advancing the concept of ___________.
a. active health promotion
b. wellness education
c. passive health promotion
d. illness prevention
87. Environmental factors that NEGATIVELY influence health include:
1) Green space in the community
2) Urban stretch and unplanned zoning
3) Disparities in socioeconomic status
4) Gang violence and delinquency
5) Organized recreation programs
a. 3, 4 and 5
b. 1, 2 and 3
c. 1, 4 and 5
d. 2, 3 and 4
88. Nurse Maribel is concerned about the rise of mosquito-borne diseases in a rural community. Effective
PRIMARY prevention for community intervention measures would include:
1) Monitoring the incidence of disease caused by mosquitos
2) Promoting ordinance that control insect breeding sites
3) Instituting education program about insect control
4) Promoting accessibility of diagnostic and treatment services
a. 2 and 3
b. 1 and 4
c. 2 and 3
d. 3 and 4
89. The amount of garbage collected in a middle-class community is huge and excessive. The best approach
the nurse can take to aid in helping decrease the amount of garbage generated is ____________.
A. promoting recycling efforts in the community
B. encouraging use of fewer polluting products
C. discouraging overuse of materials that generate trash
D. educating about overbuying household products
90. A family lives near a creek and the toilet is separated from their own house. Their excreta fall freely to the
creek. What type of toilet is this?
a. Overhung latrine
b. Bored-hole latrine
c. Antipolo latrine
d. Open pit privy

Situation
In public health, quality improvement (QI) techniques are used to improve performance and service delivery.
91. PDCA is the basic structure for most quality improvement (QI) process. In public health, PDCA is focused on
activities that are responsive to community needs and improving population health. PDCA stands for:
a. Perform-Document-Correct-Act
b. Perform-Do-Check-Analyze
c. Plan-Document-Check-Analyze
d. Plan-Do-Check-Act
92. There are several approaches and tools in (QI). One of these is the Fishbone diagram, developed by K.
Ishikawa. Which BEST describes the Fishbone diagram? It is ____.
a. A cause-and-effect diagram
b. A wholistic approach in problem identification
c. Used in interview sessions d. Useful in focusing on problem symptoms
93. As a QI tool, the Gantt chart is useful for planning and scheduling projects. Which of the following
statements is INCORRECT about the GANTT chart? It ______.
a. Provides a graphical illustration of resources
b. Plans the order in which you’ll complete tasks
c. Tracks specific tasks in a project.
d. Helps assess how long a project should take
94. The steps in conducting quality improvement consist of the following:
1. Identify and select strategies
2. Evaluate QI outcomes
3. Implement solution in small scale
4. State the problem
5. Expand scope of QI throughout the organization
Which is the correct sequence in the QI process?
a. 1, 2, 3, 5, 2
b. 2, 1, 5, 3, 4
c. 4, 2, 1, 3, 5
d. 1, 4, 5, 3, 2
95. Client satisfaction survey is an important and commonly used indicator for measuring the quality in health
in health care. In the context of public health, which is NOT a purpose of satisfaction surveys? To ______.
a. Serve as essential sources of information for identifying gaps
b. Directly address the health problems presented
c. Present the likelihood of continuing client-provider- agency relationships
d. Provide data about health-seeking behaviors

Situation
As a new nurse assigned in a municipality, you want to be a good player in health care team that you belong
to. You plan to initiate a community health project that will require interdisciplinary collaboration.
96. The attributes of collaborative teamwork are the following EXCEPT:
A. professional turfing
B. unity of purpose
C. joint responsibility
D. shared decision-making
97. In any community project, the activities of health team members are needed for success. Defining the
relationship and responsibilities and tasks of each member are important. Which stage of collaboration does
this statement refer to?
A. Commitment
B. Consensus
C. Trust building
D. Collegiality
98. Health teams have an obligation to work together to improve patient care. Which of the following
messages does NOT support this statement?
A. Use unique and complementary abilities of team members to optimize patient care
B. Forge independent relationship from other professions to advance learning
C. Recognize one’s limitations in knowledge and skills.
D. Communicate with team members to clarify each member’s tasks.
99. A member of health team proposes another community project. What is the MOST appropriate thing to do
when you disagree with the proposal?
A. Question the proposal indirectly by telling your opinions to others
B. Blame the proponent and tell him he is unprofessional
C. Be frank while remaining flexible and open-minded.
D. Be absent in the next scheduled meeting of the health team.
100. In a conflict situation, health team members have mutually reached an acceptable resolution to the issue
that partially satisfies parties concerned. This approach is referred to as ____________.
A. competing
B. avoiding
C. compromising
D. accommodating
Situation
Baby Boy Max, 6 months old, was diagnosed as having communicating hydrocephalus. Nurse Ben is planning
nursing care for this baby who is a candidate for ventriculo- peritoneal shunting.
1. When helping the parents understand the baby’s problem, which of the following should be Nurse
Ben’s ACCURATE explanation?
a. “The CSF is prevented from adequate absorption by a blockage in the ventricles of the brain.”
b. “Too much Cerebro Spinal Fluid (CSF) is produced within the ventricles of the brains.”
c. “The flow of CSF through the brain cells does not empty effectively into the spinal cord.”
d. “There is a part of the brain surface that usually absorbs CSF after its production that is not functioning
adequately.
2. Nurse Ben understands that hydrocephalus, if left untreated, can cause mental retardation because of
which of the following rationales?
a. Hypertonic CSF disturbs normal plasma concentration, depriving nerve cells of vital nutrients.
b. Gradually increasing size of the ventricles presses the brain against the bony cranium; anoxia and decreased
blood supply result.
c. Increasing head size necessitates more oxygen and nutrients than normal blood flow can supply.
d. CSF dilutes blood supply, causing cells to atrophy.
3. What is the IMPORTANT nursing care of an infant with increased intracranial pressure?
a. Check the infant’s reflexes at regular intervals.
b. Monitor the infant’s level of consciousness by stimulating frequently.
c. Weight the infant daily before feeding.
d. Elevate the infant’s head higher than the hips.
4. Baby Boy Max just had a ventriculoperitoneal shunt. His parents were worried about the prognosis.
What information should Nurse Ben give to the parents to allay their worry?
a. The shunt may need to be revised as the child grows older.
b. If any brain damage has occurred, it is reversible during the first year of life.
c. Hydrocephalus usually is self- limiting by 2 years of age and then the shunt is removed.
d. The prognosis is excellent and the valve is permanent.
5. What is the drug that will lessen CSF production that Nurse Ben will anticipate the doctor to order?
a. Spironolactone
b. Lasix
c. Mannitol
d. Diamox

Situation
Ethics is a field of moral science which deals with the morality of human acts. Registered nurses must be aware
that their actions have professional and ethical dimensions. They should strive to perform their work to the best
interest of all concerned.

6. Which of the following is NOT aligned to ethics in nursing? A Nurse ______.


a. Has the freedom to do what he likes sans responsibility
b. Is obliged to avoid what is wrong and do what is good
c. Is a person capable of knowing what is right or wrong
d. Should have a sense of accountability for his actions
7. A professional nurse has a duty to know and respect the Patient’s Bill of Rights. Which of the following
is NOT included in the Patient’s Bill of Rights. The right to/for ______.
a. A considered and respectful care.
b. Privacy and confidentiality
c. A complete and current information about his illness
d. Expect continuity of care from discharge to full recovery
8. What bio-ethical principle is violated by a nurse if he provides his patient fraudulent information about
his diagnosis and prognosis?
a. Justice
b. Autonomy
c. Beneficence
d. Veracity

9. Which of the following directly VIOLATES the Patient’s Bill of Rights?


a. Informing patients about the billing policies of the hospital
b. Disclosing the HIV result to remembers of the patient’s family.
c. Honestly telling the patient about his current condition
d. Immediately referring results of laboratory to the physician.

10. Autonomy is the prerogative of the patient to give consent or refusal of treatment with the
EXCEPTION of which of the following situations?
a. Erroneous belief of a head of a church
b. Negative effect of superstition
c. Near death
d. Peer pressure

Situation
Breastfeeding is one of the major responsibilities of OPD Nurse Mia. Along this line, she gathers pregnant women to
provide them health education about the topic.
11. Nurse Mia explains that exclusive breastfeeding is giving the baby_______.
a. Breast milk and introducing solids at one month
b. Breast milk with drops of syrup
c. Breast milk alternating with sips of boiled water
d. Only breast milk for 6 months
12. She further explains that the specific objectives of exclusive and extended breastfeeding from 2005 to
2010 are the following EXCEPT_________.
a. 50% of infants are exclusively breastfeeding up to 6 months
b. 90% of infants are started on complementary feeding by 6 months of age.
c. 70% of newborns are initiated to breastfeeding within an hour after birth.
d. Twelve (12) months is the median duration of breastfeeding.
13. A mother questioned Nurse Mia in the event that she does not produce milk, what should she do? The
BEST answer of Nurse Mia would be___________.
a. “If it is difficult on your part, then you can use local herbs said to help the production of the breast milk.”
b. “That’s a big problem ma’am, but you can still try again.”
c. “Just continue breastfeeding. This will stimulate the hypothalamus in the production of breast milk.”
d. I empathize with you. I too, do not have breast milk.”

14. Chelsea, one of the pregnant women, asked Nurse Mia to teach her the proper positioning of the baby
while breastfeeding. Among Nurse Mia’s answers, which one is NOT CORRECT?
a. The baby can lie down perpendicularly with the mother to promote better bonding.
b. The baby’s face should be towards the mother’s breast to have an eye to eye contact.
c. The baby’s head and body should be aligned to promote better swallowing.
d. The baby’s tummy should be close to the mother’s tummy for better support.
15. To convince the pregnant mothers to breastfeed their newborns upon delivery, the BEST thing that
Nurse Mia must do is to ______________
a. Explain the hospital policy on “No breastfeeding; No admission”
b. Relate her own experience on breastfeeding
c. Prepare the mothers physically, emotionally and psychologically
d. Ask other mothers to convince them to breastfeed
Situation
Jam, a mother of a 6-year-old boy Mercs has arrived at school to take her child home because the school Nurse
Miguel has verified that he has an inflamed throat. Nurse Miguel urges his mother to seek treatment because if the
causative agent is beta-hemolytic streptococcus, he may develop a disorder characterized by inflamed joints, fever,
and the possibility of endocarditis.
16. Which of the following would be the possible disorder?
a. Tetanus
b. Rheumatic fever
c. Scarlet fever
d. Influenza

17. Jam question whether her other children can catch the same disease. Which of the following should be
the nurse’s response?
a. Your other children should be taking antibiotics to prevent them from catching the same disease.
b. It is caused by an autoimmune reaction and is not contagious.
c. You appear concerned that your daughter’s disease is contagious.
d. The fact that you brought Alice to the hospital early enough will decrease the chance for her siblings getting it.
18. In addition to carditis, which of the following should the nurse assess the child?
a. Oliguria and edema
b. Malabsorption and diarrhea
c. Arthralgia and low grade fever
d. Bronchitis and pneumonia
19. After throat swab for culture and sensitivity, the nurse would expect the physician to prescribe an
appropriate antibiotic. Which of the following is the purpose of this medication? It is to prevent _______.
a. Recurrence
b. Transmission
c. Inhalation
d. Inflammation
20. If left untreated, such condition can progress to which of the following complication.
a. Kidney failure
b. Left-sided heart failure
c. Right-sided heart failure
d. Angina pectoris

Situation
Nurse Alice assists in the in-service training of nurses in St. Augustine Hospital. Based from the needs assessment,
the nurses need to be trained on patient’s safety. The first lecture was on medication administration. Nurse Alice
gave the following problems for the nurse to solve.
21. An eight-year-old- patient is to receive Aminophylline 3 mg/kg TID. If the girl weights 25 kg, how
much Aminophylline should she receive daily?
a. 125 mg
b. 225 mg
c. 200 mg
d. 100 mg
22. Mrs. Madrigal is to receive Digoxin 0.325 mg p.o. because of her heart ailment. The stock is 0.250 mg
per tablet. How many TABLETS should the nurse request for 7 days supply for the patient?
a. 9
b. 7
c. 10
d. 8
23. In giving insulin to an OB diabetic patient, the nurse must use an APPROPRIATE needle size and
length, which among the following will this be?
a. 23 G, 3/8
b. 22 G, 2/8
c. 24 G, 4/8
d. 25 G, 5/8

24. Nurse Alice emphasized that patient safety is___________.


a. Preventing errors and adverse reactions to patients associated with health care
b. Being careful in caring for patients at all times
c.Providing treatment and care to patients and documenting the same
d. Being mindful of the needs of others at all times

25. Nurse Alice has provided instructions that all care provided by nurses must be documented, as one of
their legal responsibilities. In case errors occur, which is the correct way of documenting it?
a.Wrong medicine given to wrong patient. Physician notified. Patient observed
b. Medication inadvertently omitted. Supervisor notified.
c. Medication given to patient A instead to patient B. Patient advised.
d. Wrong dose of medicine administered to patient. Sorry.
Situation
Headnurse John wishes to successfully change the way her registered nurses, nursing assistants, and other nursing
staff in providing emotional support to the ward’s patients.

26. In conducting quality improvement, Nurse John should look into which of the following as her basis,
EXCEPT:
a. Patient’s complaint data
b. Patient’s satisfaction survey
c. Incident reports
d. Patient’s chart
27. The FIRST step in the Quality Improvement process is which of the following?
a. Assess progress.
b. Examine the data.
c. Confirm the existence of the problem by gathering data.
d. Set goals.

28. The BEST quality improvement intervention that Nurse John can do is which of the following?
a. Maintain ongoing informal communication with staff.
b. Analyze patients complaint record
c. Interview staff individually and in group
d. Recommend increase in salary of nursing staff.

29. Some of the nursing staff seemed to have a sense of entitlement according to nursing department
leadership. They behaved as though they had unlimited job security (because of the strength of the
nursing unions). Which of the following action is the BEST?
a. Develop a documentation-and-tracking process for the observations
b. Revise job descriptions and performance for all nursing staff to include the new general care guideline and
highlight emotional support.
c. Continue to work on increasing staff accountability for service behaviors.
d. Develop new evidence-based general care guidelines for nurse-patient interactions.
30. Which of the following requires quality improvement? Select all that apply.
1. Patient’s fall/injury
2. Infection
3. Safety
4. Pressure ulcers
a. 1, 2, & 4
b. 3 only
c. 1 & 4
d. 1, 2, 3, & 4

31. When planning a teaching program for a child who has recently been diagnosed with type 1 diabetes,
what will be the nurse’s FIRST concern for the Adel and her parents? To let them ______.
a. Assess their own feelings about diabetes
b. Learn how to monitor blood glucose level
c. Understand why activities must be limited
d. Learn how to administer insulin injections
32. An evening snack is planned for Joyjoy receiving NPH (Novolin N) insulin. This will provide _____.
a. Encouragement for the child to stay on a diet
b. Calories to help the child gain weight
c. Nourishment to counteract late insulin activity
d. Energy for immediate utilization
33. When teaching about insulin and its potential for hypoglycemia, the nurse should include that its
PEAK EFFECT occurs in which number of HOURS?
a. 1 to 2
b. 5 to 10
c. 4 to 12
d. 2 to 4
34. When teaching Joyjoy on dietary management, what should the nurse emphasis MOST?
a. Food in the form of concentrated glucose should be available all the time.
b. Meals should be preferably prepared and eaten at home
c. Food should be weighed on a gram scale all the time.
d. Meals should be prepared separately from the rest of the family.
35. At 7 AM, the nurse receives the information that Joyjoy has a 6 AM fasting blood glucose level of 1809
mg/dL. What should be her PIORITY nursing action?
a. Inform Adel that a complex carbohydrate such as cheese should be eaten.
b. Encourage Adel to start exercising and to continue for 5 minutes.
c. Tell Adel that the prescribed dose of regular insulin should be administered.
d. Ask Adel to obtain again an immediate glucometer reading.

Situation
Joyce, 3 years old, was admitted to Pediatrics Ward because of watery stools four times in 30 minutes accompanied
by vomiting, abdominal pain and temperature of 38.1 degrees centigrade. Nurse Ali was the one who was the
admitting nurse.
36. Which of the following should be the PRIMARY consideration by the nurse when charting the Chief
Complaint? The ___________.
a. Number of significant accompanying symptoms
b. Objective symptom presented
c. Subjective symptom presented
d. Significant reason for the child’s hospitalization
37. Using the FDAR (Focus Data Action and Response) Charting, which of the following presenting data
would be PRIMARY “Focus”
a. Vomiting in small amount
b. Watery stools 4x in 30 minutes
c. Generalized abdominal pain
d. Temperature of 38.1 centigrade

38. When getting the data on the subjective symptom of the abdominal pain, which of the following
should the nurse RELY on?
a. Mother’s verbal complaint
b. Result of the abdominal percussion
c. Patient’s verbal complaint
d. Result of auscultation of the abdomen
39. The “response” should be based from which of the following categories of the FDAR?
a. Intervention made
b. Laboratory Findings
c. Focus
d. Assessment
40. Which of the following can be the LEAST source of data for this case?
a. Vital signs
b. Laboratory Findings
c. Intake and output
d. Immunization record

Situation
Hemophilia is a rare blood disorder that affects the body’s clothing factors. This may result to incapacitation
complications to children affected with this disorder. A 3-year-old child Law was diagnosed with hemophilia.
41. When counselling parents of a child who has recently been diagnosed with hemophilia, what must
Nurse Chloe KNOW about Law’s condition whose father is normal and the mother is the carrier?
a. It is likely that all sons are affected.
b. There is a 50% probability that sons will have the disease.
c. Every daughter is likely to be a carrier.
d. There is a 25% chance a daughter will be a carrier.
42. Law has slipped on the ice and bumped his knee. Which among the following should Nurse Chloe
prepare to administer, as per doctor’s order? Intravenous infusion of ______?
a. Cryoprecipitate
b. Factor VIII
c. Factor X
d. Desmopressin ( DDAVP)

43. Nurse Chloe is providing home care instructions to the mother of Law. Which of the following
complications should Nurse Thelma tell the mother, should repeated bleeding continues?
a. Leukemia
b. Hemarthrosis
c. Ecchymosis
d. Hematoma
44. A nurse analyzes the laboratory results of Law. The nurse understands that the MOST likely
ABNORMAL finding in Law is which of the following?
a. Partial thromboplastin time
b. Hemoglobin level
c. Hematocrit level
d. Platelet count
45. The nurse is planning a meal that would provide IRON for a child with bleeding disorders. Which
dinner menu would be the BEST?
a. Chicken nuggets, macaroni, peas, cantaloupe, milk
b. Fish sticks, French fries, banana, cookies, milk
c. Ground beef patty, lima beans, wheat roll, raisins, milk
d. Peanut butter and jelly sandwich, apple slices, milk

Situation
Pia was diagnosed with gestational diabetes. She is 34 years old and is on her 26 weeks Age of Gestation.

46. Gestational diabetes is said to exist in pregnancy because of the diabetogenic effect of what hormone
secreted by the placenta?
a. Human placental lactogen
b. Human chorionic gonadotropin
c. Estrogen and progesterone
d. Relaxin
47. Which of the following infection will mothers with diabetes have FREQUENTLY?
a. Moniliasis
b. Herpes zoster
c. psoriasis
d. herpes simplex
48. In 36 weeks, amniocentesis was orders by the doctor for this patient. What is the purpose of this
procedure? To detect/determined ______.
a. An increase level of bilirubin
b. Genetic abnormalities
c. Down syndrome
d. Fetal lung maturity
49. Aside from an endocrinologist, to whom should the nurse also plan to refer the patient to?
a. General practitioner
b. Neurologist
c. Surgeon
d. Hyperglycemia
50. What is the PRIMARY effect to the newborn of a diabetic mother?
a. Hypoglycemia
b. Anemia
c. hyperinsulinism
d. hyperglycemia

Situation
Nurse Cara was talking about HIV – AIDS to a group of teen-agers. Here are some questions which were asked by
them during the open forum.
51. What are some of the general symptoms the PRIMARY HIV infection? The nurse enumerated the
following. Which among these are CORRECT?
1. Fatigue 4. Sore throat
2. Headache 5. Cough
3. Fever 6. Dyspnea
a. 1, 2, 3, 5, 6
b. 1, 2, 4, 5,
c. 1, 2, 3, 4
d. 2, 3, 4, 6
52. When can AIDS be manifested? The nurse answer was. “It can be as early as _______.”
a. 1 year or as late as 2 years
b. 2 years or as late as 10 years
c. 1 year
d. 6 months
53. Nurse Cara also asked the participants if they got to know the transmission of HIV based from her
lecture? Which is Not correct?
a. Accidental blood exposure
b. Kissing
c. Unprotected sex
d. Mother to child transmission

54. On the question as to which of the following are the effects of AIDS on pregnancy, one teenager cited
a wrong answer which was ____.
a. Mild weight loss
b. Prematurity
c. Repeated abortion
d. Infertility
55. HIV transmission from mother to infant occur at post-natal period during _____.
a. Bathing
b. Washing of vagina
c. Bottle feeding
d. Breastfeeding

Situation
A 13-year-old- girl named Diana tells the nurse at the pediatric clinic that she took a pregnancy test and it was positive. She
tells the nurse that her grandfather, with whom she, her younger siblings, and her mother live, has repeatedly molested her for
the past 3 years. When the nurse asks the girl if she has told this to anyone, she replies: “Yes, my mother.”

56. As the nurse’s legal responsibility, which is the APPROPRIATE agency should she notify FIRST?
a. City Health Office to do a vaginal examination to confirm the pregnancy
b. DSWD for notifying the girl’s mother about the pregnancy test positive result.
c. Board members of Child Protective Services for proper intervention
d. Philippine National Police concerning a possible sex crime

57. If Diana is really a victim of incest and the agency opts that she be admitted in the hospital, where
should she be placed?
a. With an older friendly child
b. In a room near the nurse’s station
c. With another 5 year old
d. In a private room
58. The abusive parent usually manifests one of the following characteristics EXCEPT________
a. History of severe mental illness
b. History of drug abuse
c. Belongs to low-income group
d. Low tolerance for frustration
59. When a patient, who allegedly has been sexually abused, the nurse should be aware that legal
proceedings may be necessary. Because of this possibility, which of the following actions is MOST
important to be done by the nurse?
a. Determining if there are other injuries such as bruises and hematomas.
b. Asking the parents for their approval for diagnostic examinations for the child.
c. Assessing the child’s physical, intellectual and emotional developmental levels.
d. Documenting physical findings and interactions during admission.
60. What typical assessment findings are observed to a patient who is a victim of sexual abuse
a. Recurrent UTI
b. Fracture and bruises
c. Enuresis
d. Failure to thrive syndrome

Situation
Nurse Ella is assigned at the Under Five Clinic. Mostly of her patients are infants, so she took the opportunity to
have a mother’s class on the care of infants.

61. Which of the following will Nurse Ella EMPHASIZE regarding the importance of play during infancy?
Play enhances _______ development.
a. Cognitive
b. Emotional
c. Physical
d. Social

62. Which among the infants would show signs that he/she is experiencing maternal deprivation?
a. Hyperactive
b. Overweight
c. prone to illness
d. responsive to stimuli

63. What should Nurse Ella include in the accident prevention teaching plan for a mother with 4-month-
old infant?
a. Keep crib rails up to the highest position.
b. Cover electric outlets with safety plugs.
c. Remove poisonous substances from low areas.
d. Remove small objects from the floor.
64. When teaching a mother how to prevent accidents while caring for her 6-month-old infant, Nurse Ella
should emphasize that at this age, the infant can already do which of the following developmental
milestone?
a. Stand while holding on to furniture.
b. Crawl short distances.
c. Roll over from back to abdomen.
d. Sit up without assistance.
65. Nurse Ella assesses the oral cavity of a 6-month-old child and finds out that the tooth that buds first is
the __________.
a. Upper molars
b. Lower central incisor
c.Canines
d. Lower molars

Situation
Nurses are guided by bio-ethical principles as they practice their chosen profession.

66. What bio- ethical principle guides the nurses when taking care and being responsible for patients?
a. Solidarity
b. Fidelity
c. Totality
d. Stewardship
67. Each person has a responsibility to take care of his/her body. In case a part is defective, it can be
removed for the benefit of the remaining parts. What bio- ethical principle is this?
a. Totality
b. Stewardship
c. Solidarity
d. Fidelity

68. As member of the health team, nurses are expected to collaborate with other health care
professionals for unity and cooperation. What bio- ethical principle is this?
a. Fidelity
b. Stewardship
c. Solidarity
d. Totality
69. When another person or entity intrudes in the affairs of the patient to promote justice and welfare,
what bio- ethical principle does he/she promotes
a. Welfare
b. Stewardship
c. Justice
d. Paternalism
70. As a means to solve any crisis or scarcity in any health care systems, providing an equitable
distribution of resources is based on what bio- ethical principle?
a. Paternalism
b. Justice
c. Fidelity
d.Stewardship
Situation
Nurse Jon is caring for a mother- infant dyad. The mother has just given birth to a baby boy two days ago.
Today, the charge nurse us doing a chart audit to determine whether the nursing process was properly
done from prenatal to postnatal periods. The laboratory results of the mother and baby boy are shown
below.

71. Using the chart data, which nursing intervention was CORRECTLY done by Nurse Jon for Mrs. Mayor?
a. Rubella vaccination
b. Rhogam injection
c. Neonatal 50% glucose infusion
d. Maternal blood transfusion
72. Nurse Jon charted an IMPORTANT plan of care for Mrs. Mayor. He indicated in her charting that the
patient should report immediately signs and symptoms of a virus-causing- disease which may affect the
cardiovascular system of the fetus during which of the period of pregnancy?
a. First month
b. Third month
c. Second Month
d. Fourth Month
73. Looking at the medication sheet, the charge nurse noted that Nurse Jon used the CORRECT ROUTE in
the administration of the vaccine, which was_______________.
a. Rectal
b. Intradermal
c. Subcutaneous
d. Intramuscular

74. After administration of the vaccine, health teaching was also charted by the staff nurse. Which of the
following is the CORRECT one?
a. Immediate follow-up check up
b. Nothing Per Orem
c. No pregnancy for 3 months
d. Avoid exposure to Rubella
75. Nurse Jon should also include in his charting an additional health education to Mrs. Mayor that is:
rubella virus can be transmitted via the placenta by the process of ______________.
a. Simple diffusion
b. Pinocytosis
c. Active Transport
d. Facilitated Diffusion

Situation
Patient Stella visits her obstetrician for a positive pregnancy test. The nurse in the clinic took her history. Stella
reported that she has been pregnant four times before. She has two children at home, one of whom was born at 32
weeks gestation. She lost a set of twins at 14 weeks and another baby at 12 weeks.

76. In taking the Obstetrical score, how will Nurse Belle record Gravida, Para and (TPAL)?
a. G5, P4, T1, P1, A2, L2
b. G5, P2, T1, P1, A2, L2
c. G3, P2, T1, P2, A2, L1
d. G4, P4, T2, P3, A3, L2
77. When questioned, Stella admitted that she sometimes has several glasses of wine with dinner. Her
alcohol consumption puts her fetus at risk for which condition?
a. Alcohol addiction
b. Anencephaly
c. Learning disability
d. Down syndrome
78. To help determine whether Stella is at risk for a toxoplasmosis infection, what will the nurse ask the
client?
a. “Have you ever had osteomyelitis?”
b. “Have you recently has a rubeola vaccination?”
c. “Do you have any cats at home?”
d. “Do you have any birds at home?”
79. Stella asks Nurse Belle whether she can take castor oil for her constipation. How should the nurse
respond? “No”. it can ______.”
a. Lead to increased absorption of fat-soluble vitamins
b. Promote sodium retention
c. Produce an adverse effects
d. Initiate premature uterine contractions
80. Stella also admitted to Nurse Belle that she also use cocaine at least once per day, skipping meals at
times. Which nursing diagnosis is MOST appropriate for her?
a. Activity intolerance related to decreased tissue oxygenation.
b. Risk for infection related to metabolic and vascular abnormalities.
c. Impaired gas exchange related to respiratory effects of substance abuse.
d. Impaired nutrition: less than body requirements related to limited food intake.

Situation
Myra is now 15 years old. She goes to an exclusive school but she seldom interacts with her classmates nor is
interested to join school activities. Her mother brought her to an adolescent specialist and was diagnosed of
malnutrition.

81. Which of the following is NOT a CORRECT statement about malnutrition? Malnutrition is for _______.
a. Both the urban and rural areas
b. Both the poor and the rich
c. The underweight as family income increases
d. The overweight as family income increases
82. Advertisement (ads) on television is said to be a contributory factor for obesity. What could be a
GOOD advice to adolescents like Myra regarding these ads that can be contributory to malnutrition?
a. “Check nutritional contents before buying foods.”
b. “Follow your desire for food and drinks.”
c. “What you see is what you get.”
d. “There is no truth in advertisements.”

83. Myra went to see a nutritionist. She is overweight for her height. What is the normal weight, in
POUNDS, for Myra who is 5 feet tall?
a. 90
b. 75
c. 85
d. 80
84. How can Myra be helped to reduce her weight? She can be advised to cut intake of which of the
following?
a. Salt, protein and fats
b.Carbohydrates, vitamins and salts
c. Carbohydrates, fats and salts
d. Fats, protein and sweets
85. Which of the following can Myra do to help her with her obesity problem?
a. Skip one full meal a day.
b. Drink 6 glasses of water every day.
c. Take weight reducing pills.
d. Exercise 30 minutes three times a week.

Situation
To ensure patient safety at all times, personal and professional growth of nursing is a necessity than a choice.
Nurse Mara is a beginning nurse in one of the hospitals in the suburb. During the Orientation Program, the need to
enhance her skills, knowledge and attitude was emphasized.
86. “Commitment to continual learning and active participant in the development and growth of the
profession are commendable obligation” is contained in which of the following?
a. Code of Ethics
b. RA 10912
c. RA 7164
d. RA 9173
87. Which law declares that the policy of the State is to promote and upgrade the practice profession in
the country?
a. RA 7164
b. Ra 0173
c. code of Ethics
d. RA 10912

88. Nursing programs that are based on needs assessment and needs analysis and should be offered free
is which of the following?
a. Continuing Development Program
b. In-service Training Program
c. Nursing Education Program
d. Post-Graduate Program
89. In three year’s time, Nurse Mara needs to attain certain number of Continuing Professional
Development units to be able to renew her ______.
a. Philippine Nurses Association membership
b. Professional License
c. Professional Identification Card
d. Professional Registration
90. Nurse Mara was asked by her Headnurse why she requested permission to enroll in the Graduate
Program for the second semester. Her reply should be, EXCEPT:
a. To have an impressive resume to enable her to be competitive
b. Connect with people professionally
c. Invest for the future
d. Pursue her interest in Pediatric Nursing in more depth.

Situation
One of the responsibilities of a nurse in fertility specialist’s office is to provide health teaching to the patient in
relation to timing of intercourse to achieve pregnancy.
91. Which instruction to patient addresses the BEST time to achieve a pregnancy?
a. 14 days before the next period is expected
b. Immediately after menses end
c. Midway between periods
d. 14 days after the beginning of the last period
92. With regards to sexual functioning, the nurse explained the ovulation occurs on which of the
following conditions?
a. Chorionic gonadotropin level is high
b. Luteinizing hormone level is high
c. Oxytocin level is high
d. Progesterone level is high

93. After ovulation has occurred, the nurse teaches women in the fertility clinic that the ovum is through
to remain viable for many HOURS?
a. 24 to 36
b. 12 to 18
c. 48 to 72
d. 1 to 6
94. When teaching clients to determine the time of ovulation by taking the basal temperature, the nurse
explains that the change in the basal temperature during ovulation is shown in which of the following
observations? The temperature_____________.
a. Drops markedly and remains lower
b. Rises markedly and remains high
c. Drops slightly and then rises again
d. Rises suddenly and then falls down

95. In response to a client’s question on the chief function of progesterone, which of the following would
be the CORRECT answer? It _________________.
a. Stimulates the follicles for ovulation to occur
b.Establishes the secondary male sex characteristics
c. Prepares the uterus to receive a fertilized ovum
d. Develops the female reproductive organs

Situation
Nurse Emma made a study on domestic violence against women in a certain province in Luzon.

96. Since she wanted to capture the essence and emotion of the victims she choose to use the qualitative
design. Which among the statements below is CORRECT about qualitative design?
a. It commences study at present but consummates at any future time.
b. It allows estimation of relationship between studied variables.
c. It provides insights into attitudes, beliefs, motives and behaviors of target population.
d. It tests result through numerical data and subjects them to statistical analysis.
97. Nurse Emma plans to gather 6 participants who are victims of domestic violence where she will base
her questionnaires. What technique in qualitative research is applicable to this plan?
a. Data triangulation
b. Focus group discussion
c. Interview
d. Case study

98. To have a better analysis and interpretation of finding, Nurse Emma reviewed and compared them
with other findings of previous researches on the same topic. This is done MAINLY by doing back to
which part of the study? The __________.
a. Statement of the problem
b. Related literature and studies
c. Conceptual framework of the study
d. Theoretical framework of the study

99. Nurse Emma finally decided to make an in depth study of ONLY ONE SUBJECT of domestic violence.
What design will she use?
a. Causality Design
b. Predictive Correlational Design
c. Descriptive Correlational Design
d. Descriptive Case Study Design
100. In as much as the research may have some risks on the part of the subject who is 21 years old, Nurse
Emma must assure the subject of her anonymity, confidentiality and respect for her human rights. ONE
ABSOLUTE WAY is to get an informed and written consent from whom?
a. Parents
b. Guardians
c. Husband
d. Subject herself
Situation 1
Because it is already her board exams in a few days, Tamara practices computing medications dosages with the
following scenarios.
1. Nurse Ailah administered 2 tablets of Losartan to Patient Grande within the past 24 hours. According
to the doctor’s orders, Patient Thea is to receive 100mg of valsartan a day. With this information, Tamara
knows that the stock dose Nurse Ailah has on hand is:
a. Losartan 1 mg/tablet
b. Losartan 25 mg/tablet
c. Losartan 20mg/tablet
d. Losartan 50 mg/tablet
2. Phenytoin (Dilantin) 200mg PO BID was prescribed to a patient. The medication label reads 0.1
g/capsule. For each dose, the nurse prepares how many capsules to give to the patient?
a. 500
b. 2
c. 3
d. 4

3. Mannitol 0.25g/kg/day was prescribed to a 7-year-old patient with meningitis. The patient weighs 28.6
lbs. How much mannitol should the nurse give?
a. 32.5 g
b. 3.25 mg
c. 3.25 g
d. 0.325 g
4. Nurse Ailah received an order of enalapril 10 mg OD for a hypertensive patient. How many tablets will
she give to the patient if they have a stock dose of 20mg/tablet?
a. 1/2 tablet
b. 1 tablet
c. 1 ½ tablets
d. 2 tablets
5. Amoxicillin 250 mg oral suspension is prescribed to a patient q8h. Nurse Ailah checked the stock dose
and saw that the bottle reads Erythromycin Oral Suspension 125mg/5mL. How much will Nurse Ailah
administer to the patient in a day?
a. 6 mL
b. 12 mL
c. 10mL
d. 30mL

Situation 2
Nurse Ravi is reviewing concepts and practicing skills on intravenous transfusions.
6. Nurse Ravi is selecting a vein for an intravenous transfusion in a 62-year-old patient. He wants to
dilate and distend the veins for easier insertion. He does the following methods, except:
a. Tapping the selected vein multiple times
b. Placing extremity in a dependent position if there are no contraindications
c. Applying warm washcloth on to the extremity for several minutes
d. Stroking the extremity from distal to proximal below his desired site

7. Which among the following catheters is used for less than 24 hours?
a. Butterfly needle
b. Midline catheter
c. Plastic indwelling cannula
d. Angiocath
8. How often should Nurse Ravi observe a patient with intravenous infusions?
a. Every 2 hours
b. Every 6 hours
c. Every 8 hours
d. Every 12 hours
9. Six hours after initiating an IV line in a patient, Nurse Ravi noticed that the IV site was red and warm.
The patient complained that it was painful. Nurse Ravi would suspect which of the following?
a. Phlebitis
b. Extravasation
c. Infiltration
d. Clot formation

10. What should be the size/gauge of the needle Nurse Ravi would use in administering IV infusions for
trauma patients?
a. 18
b. 22
c. 20
d. 24

Situation 3
Matthew rushed to the ER due to severe abdominal pain radiating to the scapular region. Health history revealed
that his meals were mostly from KFC. Patient appeared jaundiced with facial grimaces. The physician on duty
(POD) examined him and an impression of Cholelithiasis was given by the POD. Patient was ordered to be admitted
for further work-up and possible surgery.
11. Which of the following questions should be asked by the nurse to accurately describe biliary colic?
A. Nararamdaman niyo po ba ang sakit bagomatulog?
B. Nararamdaman niyo po bang sakit sa kaliwang parte ng inyong tiyan?
C. Panandalian lamang po ba ang sakit?
D. Nararamdaman niyo po ba ang sakit matapos kumain?
12. In a patient with obstructive jaundice, which of the following assessment findings would you expect?
a. Itchiness of the abdomen
b. Clay-colored stool
c. Reddish-colored urine and stool
d. Presence of urobilinogen in the urine
13. Mr. Matthew bile duct exploration with open cholecystectomy. A t-tube was connected to drain after
surgery. After 12 hours, there was a marked decrease in output from 260 ml to 90 ml. What is the nurse’s
priority action?
A. Call the physician stat
B. Assess the tube for any obstruction
C. Irrigate the tube with normal saline
D. Change the position.

14. One of the most common complications post-operatively is:


A. Pleurisy
B. Bronchopneumonia
C. Pneumonia
D. COPD
15. Which dietary restriction should be recommended by the nurse to help prevent recurrence of
stone formation?
A. Orange juice, pan de sal
B. Whole milk, dairy crème , fried chicken breast
C. Steamed lapu-lapu, nilagang okra, salabat
D. Nilagangsaba, vegetable salad,

Situation 4
Mr. Sonny, a 62 year-old male, complains of shortness of breath, dyspnea on exertion, palpitation, and
expectorating frothy, blood-tinged sputum. He was brought to the emergency room of Hospital C. During the
interview, he also complained of easy fatigability and weakness.

16. Given the following assessment cues, Nurse Olivia makes a nursing care plan with the priority
nursing diagnosis of:
A. Anxiety related to actual threat to biological integrity secondary to heart failure
B. Ineffective breathing patterns related to decreased respiratory depth secondary to pain
C. Activity intolerance related to compromised oxygen transport system secondary to heart failure
D. Activity intolerance related to compromised oxygen transport system secondary to heart muscle dysfunction
17. Mr. Sonny has been diagnosed with heart failure. Nurse Olivia expects this patient to not exhibit the
following signs and symptoms EXCEPT:
A. Pulse rate of 48 bpm
B. Murmurs
C. Respiratory rate of 18 cpm
D. Strong, bounding pulse

18. The doctor ordered oxygen administration for the patient. The nurse knows that the rationale
behind this intervention is that:
A. Oxygen helps to decrease the work of breathing
B. Oxygen is contraindicated in heart failure so the order is questionable
C. Oxygen is given to reduce anxiety
D. Oxygen is administered through mechanical ventilation only

19. Nurse Olivia prepares to perform the following interventions for the immediate management of
Mr. Sonny excluding:
A. Establish an IV line and administer PNSS immediately
B. Monitor heart rate and dysrhythmia by using a cardiac monitor
C. Continuously assess level of consciousness
D. Provide reassurance and support to the patient
20. The nurse knows that the management of a patient with heart failure requires interprofessional
collaboration. Mr. Sonny asks Nurse Olivia from whom can he ask for assistance and counsel regarding
home care services. Nurse Olivia responds that the professional who can help him with his concern and
who assists the continuing care nurse with planning the patient's discharge is the:
A. Physical therapist
B. Social worker
C. Psychologist
D. Head nurse

Situation 5
Nurse Queen Diana is a staff nurse in the medical ward. Most of her patients are afflicted with intestinal and rectal
disorders.
21. Upon assessment, which of the following differentiates Crohn’s disease from ulcerative colitis?
A. Presence of bleeding: severe in Crohn’s disease while in Ulcerative Colitis bleeding is mild.
B. Presence of diarrhea: severe in Crohn’s disease in ulcerative colitis it is mild
C. Affected area: Crohn’s disease is the descending while Ulcerative colitis is the ascending colon.
D. Course of the disease: Crohn’s disease is prolonged and variable, Ulcerative colitis has a remission and
exacerbation

22. In terms of therapeutics, which of the following medications can be used for inflammatory bowel
diseases?
A. Corticosteroids
B. Atropine sulfate
C. Dulcolax
D. Maalox
23. Which systemic complication can be expected from a patient with Crohn’s Disease?
A. Perforation
B. Small bowel obstruction
C. Hemorrhage
D. Megacolon

24. In order to accurately diagnose ulcerative colitis, which of the following laboratory studies is
needed?
A. Proctosigmoidoscopy
B. Albumin study
c. colonoscopy
d. ultrasound
25. Which of the following is the MOST appropriate nursing diagnoses for a patient in acute
exacerbation of ulcerative colitis?

i. Imbalanced nutrition less than body requirements R/T impaired absorption

ii. Risk for deficient fluid volume R/T abnormal fluid loss

iii. Risk for ineffective tissue perfusion R/T low hemoglobin

iv. Acute Diarrhea R/T inflammation of the bowel


a. ii& iii
b. I, ii, & iii
c. I, ii, iii, & iv
d. i& ii

Situation
In her 15 years of being a ward nurse, Vebs has witnessed and taken cared of several terminally-ill patients and
their families.

26. Which among the following is not considered palliative care?


a. Administering anti-emetics to a cancer patient who underwent chemotherapy
b. Frequently turning the position of a comatose patient post-CVA
c. Removing the tumor in a Stage I breast cancer
d. Providing pain relief to a cancer patient
27. What is the immediate focus of nurses in caring for a grieving family who just lost their father from
cancer?
a. Post-mortem care
b. Return patient’s belongings (watch, clothes, etc) to the family
c. Obtaining consent for autopsy
d. Provide and facilitate time for mourning

28. Which among the following is not true regarding grief in the older population?
a. Many older adults exhibit resilience in facing losses.
b. They are not at risk for complicated grieving.
c. Their increased age increases their likelihood of having faced multiple losses.
d. Positive reappraisal helps older adults adapt to significant losses.

29. Which among the following statements from Nurse Vebs assesses the social support systems of a
person experiencing grief from the death of his sister?
a. “What does this loss mean to you?”
b. “Tell me how you are feeling.”
c. “Which friends or family members do you wish were here with you?”
d. “Tell me about your relationship with your sister.”
30. Which nursing action of Nurse Vebs will promote a dying, elderly patient’s dignity and self-
esteem?
a. Providing him regular baths and good hygiene
b. Instructing the patient’s family members to decide for his food choices
c. Calling the elderly patient by his first name only
d. Encouraging his siblings to join during your private conversations with the patient

Situation
Nurse Nikki handles various patients with elimination problems. She always performs her best nursing care to
these patients.

31. Nurse Nikki teaches an incontinent patient bowel training. Which among the following would
Nikki not include in her health teaching?
a. Opioids are avoided to prevent decrease in bowel peristalsis.
b. Regular exercise is encouraged.
c. Leaning backward at the hips while sitting on the toilet will help stimulate colon emptying.
d. A hot tea or fruit juice may be taken before the patient defecates.
32. An enema to provide relief from gaseous distention is prescribed to a patent. Nikki knows that this
is a/an:
a. Oil retention enema
b. Carminativeenema
c.Soapsuds enema
d. Fleet enema

33. After inserting a nasogastric tube, Nikki checks its placement by measuring the pH of the aspirate.
Nikki knows that proper placement has likely been made if the aspirate has a pH of:
a. pH 3.0
b. pH 7.0
c. pH 9.0
d. pH 7.4

34. Nurse Nikki knows that the approximate length of a normal female urethra is:
a. 1.5-2.5 cm
b. 4-6.5 cm
c. 20-25 cm
d. 10-12 cm
35. To prevent urinary tract infection in a female patient with an indwelling catheter, Nikki instructs
her to decrease intake of which of the following fruit juices?
a. Pineapple juice
b. Prune juice
c. Apple juice
d. Cranberry juice

Situation
Anna, while going on a date with Karl, suddenly complains “I can’t breathe”. Karl rushed her immediately in the ER.
She was given Theophylline and Cromolyn sodium for the attack.
36. The following are expected assessment findings for Anna except:
A. Decreased tactile fremitus
B. Resonant to tympanitic
C. Wheezing sounds
D. Crackles

37. Upon administration of theophylline, you are monitoring Anna for which drug induced adverse
effect?
A. Bradycardia
B. Diarrhea
C. Constipation
D. Restlessness
38. Upon administration of Theophylline, the therapeutic effect that you would expect is:
a. Decrease in breath sounds
b. Increase in body temperature
c. Decrease in wheezing
d. Pruritus
39. Karl asks the nurse why Anna needs Theophylline. The best response of the nurse is:
a. “Pinaluluwag nito ang daluyan ng hangin.”
b. “Iniiwasan nito na magkaroon pa ng allergic attack si Athena.”
c. “Iniiwasan nito na atakihin ng epilepsy si Athena”
d. “Binabawasan nito ang pamamaga ng baga”

40. Karl brought food in Anna’s room. Which of the following food should be restricted for Anna in
consideration to her therapeutic regimen?
a. White Chocolate Mocha drink
b. Mango juice
c. One piece chicken with rice
d. One peach mango pie

Situation
Nurse Wanda has newly admitted patients in the Medicine Ward who are for workup.
41. A patient with suspected pneumonia is ordered to have sputum culture and sensitivity. Nurse
Wanda orders the patient to cough up how much sputum into the container?
a. 15-30mL
b. 1-5 mL
c. 4-10mL
d. 250mL
42. Luke is a 25-year-old patient suspected to have chronic myelogenous leukemia. He is advised to
undergo bone marrow biopsy. The following bones may be used for the procedure, except:
a. Posterior superior iliac
b. Anterior superior iliac spine
c. Sternum
d. Triquetrium

43. Patient Sola likes to eat melons and strawberries. On the day of her specimen collection for fecal
occult blood, she claimed that she has been bingeing on these fruits since 5 days ago. What will be its
implication on the test/results?
a. Results can become falsely positive for occult blood.
b. Results can become falsely negative for occult blood.
c. Increased intake of fruits will confirm presence of occult blood in stool.
d. Stool specimen will become clay-colored.

44. Nurse Wanda knows that 24-hour urine collection is needed in determining the following, except:
a. Vanillylmandelic acid (VMA) in pheochromocytoma
b. Uric acid in gouty arthritis
c. Culture and sensitivity of causative agent in urinary tract infections
d. Creatinine clearance in acute kidney injury (AKI)
45. Immediately after obtaining blood for an arterial blood gas examination, Nurse Wanda instructs
the patient to do which of the following?
a. Perform ROM exercise on the proximal joint.
b. Apply pressure for 5-10 minutes
c. Elevate extremity above heart level.
d. Increase oral fluid intake.

Situation
The nursing director assigned Xavier, RN in the orthopedic ward to care for patients with mobility problems.

46. Xavier is caring for a patient with an arm cast. He knows that the functional position of the wrist
and fingers is which among the following:
a. Wrist in extension, fingers and thumb in flexion
b. Wrist in flexion, fingers in flexion, thumb in extension
c. Wrist in flexion, fingers and thumb in flexion
d. Wrist in extension, fingers in flexion, thumb in extension
47. Xavier is teaching a patient with crutches how to go up the stairs. Which among the following
should he instruct the patient with?
a. When moving the affected leg up, the body weight should be on the crutches.
b. The unaffected leg goes up the step after the affected leg.
c. Patient should keep his feet together at the bottom of the stairs
d. The crutches and the affected leg go up the step together.
48. Xavier is assisted by another staff nurse in lifting a patient. Which among the following actions will prevent injury to
Xavier?
a. Keeping the feet wide apart and knees extended
b. Positioning himself close to the patient
c. Using the muscles of the back in lifting
d. Rotating at the waist in lifting
49. Nurse Xavier asks two more nurses to help him logroll a patient. Where should Xavier place the patient’s arms prior to
logrolling?
a. Along the sides
b. Crossed on the chest
c. Overhead, extended
d. Behind the hips
50. In applying restraints, Xavier should employ which type of ie?
a. Half-bow
b. Surgical tie
c. Double knot
d. Two-throw knot

Situation
Nurse Melody admits Mr. Ong, 34 year-old, who was brought to the Emergency room by his co-worker because of
severe abdominal pain. He was observed to be very anxious; holding his abdomen with his right hand while
bending towards his right side. The Emergency Room doctor made his initial assessment and referred the patient
to the Surgical doctor on duty for further assessment.

51. To describe a sensation of severe pain felt for ruptured appendicitis, Nurse Melody applies deep
pressure of her finger into the abdomen at a point away from the pain and then releases. This maneuver
is termed as __________.
A. Perforation
B. McBurney’s Point
C. Rovsing’s sign
D. Rebound tenderness
52. Mr. Ong was diagnosed with ruptured appendicitis. When the appendix is suspected ruptured, a
generalized abdominal pain is felt and becomes ____________.
A. Merely a discomfort and steady rather that intermittent
B. Intense and the patient guards the area by living room
C. Rigid, stiff and board like feeling
D. Tender and focused on the umbilical cord region
53. Immediate surgery was done on Mr. Ong. Which of the following measures for pain control is NOT
advisable to be done by Nurse Melody while patient Ong is having post-op pain?
A. Coughing exercises
B. Deep breathing
C. High-fowler’s position
D. Semi-fowler’s position
54. After operation, a nasogastric tube (NGT) was inserted to Patient Ong. The rationale for this surgical
measure is to _________.
A. Prevent use of antibiotics
B. Decrease hospitalization to two days
C. Decrease pain after the operation
D. Avoid the use of analgesics

55. Nurse Melody assesses the pain intensity of Mr. Ong. The pain assessment tool APPROPRIATE to be
used for Mr. Ong is _____________.
A. Faces Pain Rating scale
B. Visual Rating Scale
C. World Rating Scale
D. Numeric Rating Scale

Situation
You are the nurse on night duty in the medical ward. You were assigned by Ms. Love, the Charge Nurse of the unit to monitor
the status of a 9 year old boy with bronchial asthma.
56. Which of the following manifestations of the child would cause you to be alarmed?
A. Excessive sweating
B. Choking sensation
C. Cough with phlegm
D. Generalized chest tightness
57. You would like the child to be comfortable in bed. Which position should you put the child to ease his
discomfort?
A. Lying on his side
B. Prone position
C. High Fowler’s Position
D. Supine position

58. The Pediatrician ordered theophylline by IV route. The effect(s) of this drug is __________.
A. Decrease gastric acid secretion
B. Causes coronary vasoconstriction
C. Relaxes the smooth muscle of the bronchial tree
D. Stimulation of the peripheral nervous system

59. What APPROPRIATE activity should you advise the child while in the hospital?
A. Talk on the phone with his classmates adolescent
B. Finish his assignments in school promote rest
C. Watch cartoon movies on TV
D. Play with his teddy bear promote rest, reserve energy
60. The patient is ready to go home. What health teachings can you give to him and to his mother?
1) Prevent the child from exposure to dust and pollutants
2) Avoid exposure to extreme hot or cold weather
3) Increase fluid intake
4) Avoid hyper-allergenic food
A. 1 and 2
B. 2, 3, 4
C. 1, 2, 3, 4
D. 1 only

Situation.
Cassandra, 65 year-old cook, restless and obese was brought to the Emergency Room due to right abdominal pain,
nausea and vomiting for a week. She frequently changes her position to a more comfortable one due to intense
pain. You are the nurse in charge who admitted the patient after the attending doctor diagnosed her with Acute
Cholelithiasis.
61. Patient Cassandra’s vomiting can be treated with a nasogastric tube insertion with a suction
attachment. This measure will remove the gastric juice stimulating painful contractions of the gallbladder
which is __________.
A. Cholecystokinin
B. Ecchymosis
C. Lithotripsy
D. Acalculus
62. Patient Cassandra, was ordered by the physician to undergo several diagnostic tests. You expect that
patient will have an elevation of ________.
A. Bromsulphalein excretion
B. Lactate dehydrogenase
C. Serum gamma glutamyltranspeptidase
D. Alkaline phosphatase
63. Nurse Edwin provides important instructions to Ms. Cassandra on the prevention of Cholelithiasis.
Select all that apply.
1) Maintain ideal weight
2) Adapt low fat diet
3) Regular exercise
4) Prevent pregnancy
A. 2, 3 ,4
B. 1, 3, 4
C. 1, 2, 3
D. 1, 2, 4

64. Ms. Cassandra is prepared for surgery. Nurse Edwin provides comfort measures to relieve pain with
the following nursing intervention, EXCEPT _______.
A. Quiet environment
B. Positioning
C. Administering analgesic
D. Back rub

65. Immediate postoperative care of Nurse Edwin includes providing careful instructions to Ms.
Cassandra. Which of the following interventions should be delegated to the caregiver?
A. Administration of intravenous therapy
B. Reinforce information on exercise
C. T-tube care and changing
D. Instructions for the reduction of pain

Situation
Miles, 40 years old, came to the hospital for a diagnostic work-up for a cholelithiasis prior to surgery.

66. Which of the following clinical manifestations would you MOST likely to observe when a patient is
suffering from Cholelithiasis?
A. Indigestion, pain on the right upper quadrant radiating pain to the shoulder
B. Vague left upper quadrant pain radiating to the scapula
C. Feeling of abdominal tenderness after waking-up in the morning
D. Feeling of abdominal fullness after taking meals
67. Which ONE of the following vitamins is interfered when there is obstruction of the bile flow in
cholecystitis?
A. Vitamin C
B. Vitamin B
C. Vitamin B12
D. Vitamin K

68.The patient was ordered Endoscopic Retrograde Cholangiopancreatography (ERCP). He was put on a
mild sedation. As a Nurse, what should you check as a PRIORITY action when he returns to the ward?
A. Irritation of oropharynx
B. Nausea and vomiting
C. cough and gag reflex
D. Signs of bleeding
69. Which of the following conditions is NOT a side effect of anticholinergic drugs?
A. Constipation
B. Blurred vision
C. Dryness of the mouth
D. Increased blood pressure
70. When a client has a nasogastric tube (NGT) due to abdominal distention post surgery. Which of the
following nursing responsibilities should you do?
1) Measure gastric secretion every shift
2) Observe and record color, viscosity of secretions
3) Ensure that the gastric tube is patent
4) Assist patient to assume a comfortable position
A. 1, 2, 3 and 4
B. 1, 2 and 3
C. 1 and 2
D. 2, 3, and 4

Situation
The hospital director of a tertiary hospital is alarmed with the recent report of the head Human Resource
Department Office (HRDO) that there is so much conflict going on among the health care professionals which is
affecting quality patient care.
71. Which of the following is a form of dispute resolution and considered a legal technique which is
outside the court?
A. Suppression
B. Arbitration
C. Bribe
D. Negotiation

72. Change is defined “as making something different from what it was”. If you become a unit manager in
the future, which of the following is your FIRST step as an agent in promoting successful change in your
health care setting?
A. Revisit Institutional Vision-Mission and adhere to its commitment
B. Obtain majority agreements when issues are presented
C. Include obsessed people as part of your Task force for change
D. Ignore organizational conflict as possible

73. Decision-making in an organizational structure comes at many levels. The level which is considered
empowering to staff-member is when the unit manager does which one of the following?
A. Makes a decision when she reads the reported concerns
B. Ask direct report from personnel to decide on a solution
C. Makes a decision after convening a core group to analyze the concerned issues
D. Makes a decision by herself and assumes responsibility for her action
74. The nurse manager is receiving some feedback that one of her staff-nurses has been observed to be
always irritated, “bossy”, and indifferent to her patients and some co-workers. Which of the following
PRIORITY nursing actions should the manager implement?
A. Conduct a joint meeting of both parties involved
B. Request the concerned patients and co-workers to submit an Incident report
C. Call the nurse concerned and talk to her privately about the observations
D. Give a memo to the nurse involved
75. A team building session was conducted by the HRDO task force to come up with strategies for conflict
resolution. Which of the following is a WIN-WIN approach in conflict management?
A. Each party involved agree to respond to majority vote
B. Each participants agree to resolve conflicts by adhering to the Institutional Vision and Mission
C. Each participants will give way to 60-40% of their agreements
D. Disagreements are settled with respective compromises on issues presented
Situation
Emily, a retired Barangay Health Worker, came to the OPD for her check-up for her diabetes mellitus. She had been
diabetic since she was 37 years old. She has been taking her maintenance medications which she sometimes does
not comply with.

76. There are metabolic abnormalities in the development of type 2 Diabetes. Which of the following is
NOT included in these abnormalities?
A. Inappropriate production of the liver
B. Increased ability of the pancreas to produce insulin
C. Insulin resistance
D. Altered production of hormones by adipose tissues
77. Ms. Emily was admitted to the hospital for further check-up. Which of the following diagnostic tests
do you expect to be ordered by the diabetologist as an indicator that the patient is compliant to her
prescribed diet?
A. Oral glucose tolerance test
B. Glycosylated hemoglobin level (hgb, Alc)
C. Finger glucose findings for one day
D. Fasting blood glucose level
78. While Nurse Denise was completing her assessment, she discovered the following findings. Which of
the following should she refer immediately to the physician?
A. Tingling sensation of the hands and feet neuropathy
B. Changes in the peripheral vision retinopathy
C. Beginning ulceration of the left big toe
D. Fruity odor breath

79. Nurse Denise, the nurse in charge of patient Emily, informed her physician that her serum glucose
level is 38mmol/L and quite unresponsive to verbal questioning. The nurse suspects that she is starting
to develop Diabetes Ketoacidosis (DKA). Which of the following manifestations is UNIQUE to this
condition?
A. Shallow slow respirations
B. Increased serum potassium
C. Rapid deep respirations
D. Decreased serum albumin
80. Nurse Denise’s counseling role includes lifestyle changes as well as pharmacologic regimen. Emily’s
family were interested to know information regarding insulin. She differentiated an intermediate acting
insulin from that of short-acting which is _________.
A. Regular onset is 2 hrs. Peak is 3 ½ hr., duration -7 hrs., administered 20-30 min. before meal
B. Regular onset is 2-4 hr., peak is 4-12 hr., duration is 8 hr., administered 20-30 min. after meal
C. Regular, onset is 1 ½ hr, peak is 3-4 hrs, duration is 6 hrs. administered 20-30 min after meal
D. Regular onset is 1 ½ - 1hr, peak is 2-3 hr., 4-6hr duration administered 20-30 min. before

Situation.
Nurse Tyler admitted Darlyn, 38 years old, who was diagnosed by her physician with Type I Diabetes Mellitus. She
complains of easy fatigability and increased tiredness. Nurse Tyler, during the assessment, noted the presence of a
foot ulcer which Darlyn was trying to hide.
81. Nurse Tyler holds the hands of Darlyn while doing her assessment. Her hands were cold and clammy,
she suspected that the patient was having hypoglycemia. To relieve her condition, an immediate action is
to offer her _________.
A. Candy
B. Full meal
C. Sandwich
D. Fruits
82. Nurse Tyler watches Ms. Darlyn while sleeping. An EXCELLENT guideline to differentiate if a patient
is just sleeping or experiencing hypoglycemic reaction is when one is observed to be _______.
A.With dyspnea
B. Snoring
C. With diaphoresis
D. Grunting his teeth

83. Nurse Tyler practiced surgical asepsis during foot care. Which of the following techniques has to be
observed?
A. Opening container of sterile water after putting gloves on.
B. Changing the sterile field after sterile water spilled on it
C. Placing sterile dressing 2 cm from the edge of the sterile field.
D. Cleaning the wound in circular motion from outer towards the ulcer site
84. Nurse Tyler after foot care encourages Patient Darlyn to do which of the following APPROPRIATE
measures?
A. Apply lanolin generously between the toes
B. Cut toenails rounded at the edges and close to the skin
C. Apply alcohol after cleansing feet to improve infection
D. Wear a shoe-type slipper at home
85. Ms. Darlyn was administered NPH insulin at 8 am. At what time would Nurse Tyler expect the patient
to be at risk for hypoglycemic reaction.
A. 11 am
B. Noon
C. 6 pm
D. 4 pm
Option 5

Situation
A 6-year old boy is suffering from pain on his right inguinal region. His mother brought him to the hospital and was
diagnosed to have incarcerated hernia. He had a scheduled herniotomy.

86. What will be your FIRST pre-operative nursing measure?


A. Get informed consent from the mother.
B. Change the boy’s street clothes to surgical gown
C. Shave the operative site
D. Call the OR nurse for scheduling
87. The boy is transferred to the operating room. What is the BEST mode to be used in transferring the
patient?
A. Nurses’ carry
B. Stretcher
C. mother’s carry
D. wheelchair
88. The nurse must prepare the OR environment to be safe and comfortable for the child by reducing the air conditioning
temperature so as to prevent ___________.
A. Hypoventilation
B. Hyperventilation
C. Hypothermia
D. Hyperthermia
89. The scrub nurse has to prepare for a pediatric herniotomy. Which of the following should be ready?
A. Subcuticular 3/0 round, absorbable
B. Subcuticular 3/0 cutting, absorbable
C. Monofilament 3/0 cutting, absorbable
D. Monofilament 3/0 round, absorbable
90. The mother was afraid to submit the child for surgery. What is the BEST response to the Nurse in this
situation?
A. “It is alright to feel that way but you cannot do anything now.”
B. “It is the doctor's order; you’ll have to accept the situation.”
C. “Your child’s condition is an emergency case. If he is not operated on, he will die.”
D. “I understand your feelings, but your child needs to be operated on.”

Situation.
There are extensive legal aspects that guide the Filipino nurses in terms of regulation in practice of her profession.

91. The Professional Regulation Commission, Board of Nursing (PRC– BoN) as a regulatory board has the
legal functions and responsibilities EXCEPT in ONE of the following ___________.
A. Ensure the quality nursing education by jointly monitoring with the Commission on Higher Education (CHED)
adherence to its guidelines, policies and standards
B. Monitor and enforce quality standards and nursing practice in the Philippines to ensure efficient, ethical, moral,
technical professional standards
C. Issue, suspend or revoke certificates of registration for the practice of nursing
D. Recognize specialty organizations in coordination with the accredited professional organization and collect fees
to upgrade the conditions of Nurses
92. You are a nurse in Las Piñas Doctors Hospital and have just completed a 2 year experience in the
surgical unit. One day, your supervisor ordered you to go on a float to the Intensive Care Unit (ICU) as
two of the nurses called “on sick” in the said unit. What is the MOST important action you should do in
this situation?
A. Explain to your supervisor that you might not be able to perform safely your duty as a nurse in the unit
B. File a leave instead and call your husband to fetch you in the hospital
C. Refuse the order and tell your supervisor you don’t want to put your license in jeopardy
D. Report to the surgical ward of your assignment and request one of your nurses to replace you instead as floater
in the ICU.

93. Janna, a 35-year old, married to an insurance agent, was diagnosed with colon cancer only a month
ago. The couple have very limited information about this condition. One day, you saw the husband in the
room reading the chart which was inadvertently left by a medical intern. What PRIORITY Nursing actions
should you do in this situation?
A. Get the chart from the husband so he will not be able to read the diagnosis of his wife
B. Explain to the husband as per policy, relatives are not allowed to read the chart
C. Report immediately to the supervisor on duty and submit an incident report
D. Ask the husband what information he would like to ask from the health care professional regarding medical plan
and diagnosis
94. Nurse Nicole, is assisting Mr. Caca who just came from the Recovery unit for a thoracic surgery. She
noted that the dressings are saturated with blood, but the Surgeon just reassured the Nurse to reinforce
the dressings and monitor him closely. After an hour, Mr. Caca went into shock and died. The family filed
charges against the Doctor and the Nurse. What is the lawsuit involved in this case?
A. Murder
B. Assault
C. Battery
D. Malpractice
95. What is the MOST important Nursing measure that Nurse Nicole should have done to avoid this
situation?
1) Monitor and record closely the vital signs until stable
2) Report to the surgeon the vital signs and degree of bleeding from operative site
3) Call the supervisor on duty to assist you in referring the patient to the surgeon
4) Ask the family members to call the surgeon to see the patient
A. 2 & 3
B. 1, 2 & 3
C. 1 & 2
D. 1 & 3

Situation
Honey, 23 year old, a dental student has been having frequent bouts of tonsillitis in the past. One day, she
developed low grade fever, dyspnea, dizziness, syncope, and fainting spells while preparing for her final
examination. She was rushed to the hospital by her parents and was seen in the Emergency room by the physician
on duty (POD), Ms. Jenny was the nurse-on-duty. The POD ordered a 12 lead Electro-cardiogram (EKG) and other
laboratory tests.
96. What specific information is essential for the Health professionals to know when this EKG is ordered
for patient Honey?
A. Rate, rhythm and pressure within the blood vessels
B. Rate, rhythm, axis and circulation
C. Rate, rhythm, axis, hypertrophy, and infarction
D. Rate, hypertrophy and blockage to the chambers of the heart

97. Aside from the EKG and laboratory studies, the POD ordered patient Honey to undergo cardiac
catheterization to establish further the medical diagnosis of Valvular heart disease. Which of the
following is NOT true as part of Nurse Jenny instructions to the patient regarding this procedure?
A. Signing of consent is optional
B. Normal procedure is less than 2 hours
C. Fasting should be 8 to 12 hours
D. Presence of pounding sensation in the chest during the procedure

98. Nurse Jenny must be alert that certain complications may arise after the procedure. What PRIORITY
nursing assessment should she observe?
A. Tingling sensation
B. Signs of bleeding at the puncture site
C. Blurring vision
D. Disorientation
99. When teaching patient Honey about self-management following cardiac catheterization, Nurse Jenny
should emphasize to AVOID which ONE of the following?
A. Diet high in Protein and Carbohydrates to low cholesterol and fats
B. Lifting objects heavier than five (5) lbs
C. Taking shower with the site covered after 24 hours
D. Light exercise program within 48 hours when tolerated

100. The POD established Honey’s diagnosis that she is suffering from acquired valvular heart disease.
This condition is brought about by _______.
A. Rheumatic fever
B. Drug abuse
C. alcohol consumption
D. heredity
Situation
The assistant chief nurse of X hospital attends a seminar on quality and performance improvement. The seminar is
to increase awareness on how performance improvement affects client care and the health care organization.
1.Which of the following principles of quality improvement is MOST appropriate for patient care?
a.The priority is to benefit clients and all other internal and external customers.
b.Improvement of the quality of service is a continuous process.
c.Improvement opportunities are developed by focusing on the work process.
d.Quality is achieved through the participation of everyone in the organization.

2.The assistant chief nurse understands that the implication of quality improvement for client care can be
measured by the overall value of care. Outcomes can be measure by which of the following?
a.Client’s recommendation
b.Clinical out come
c.Cost of care
d.Client satisfaction
3.The assistant chief nurse is oriented on the various improvement strategies at the organization level.
One of these strategies is benchmarking. Which of the following describes the bench marking process?
a.Comparing data on the literature with the data collected per client.
b.Reviewing own unit’s data for opportunities.
c.Collecting data of the individual client.
d.Comparing data with that of other organization to identify opportunities.

4.The assistant chief nurse understands that a sentinel event review is one strategy to improve the health
care delivery system. The FIRST action to be initiated following a sentinel event is for the assistant chief
nurse to ____.
a.Conduct an immediate investigation
b.Conduct a root cause analysis
c.Recommend corrective action of personnel involved
d.Recommend what action personnel involved should avoid

5.Quality improvement can be achieved through the participation of everybody in the health care
organization at all levels. Which of the following is an example of nurses’ particiption as an individual?
a.Work with others in the unit to change the way client’s report is given to be more time efficient.
b.Suggest the process for notifying the pharmacy department about a missing medication.
c.Participate in a term to find a solution.
d.Change the activities of her day to spend more time with a client’s family.

Situation
The nurse assists in the care of a female client, 45 years old admitted for severe pain related to cancer.
6. In relieving pain related to cancer, which of the following nursing actions is MOST appropriate?
A. Keep the room well-lighted so that the nurse can assess the client thoroughly.
B. Allow the client to stay in one position to prevent the occurrence of pain.
C. Apply heat or cold in the areas that are painful as prescribed by the physician.
D. Place a hand bedroll behind the client’s back.

7. The client has a tunneled epidural catheter to control pain. The catheter site should be assessed every
shift by the nurse on duty. Which of the following signs indicate catheter migration or tissue trauma?
A. Bright red bleeding under the dressing.
B. Catheter insertion site is red, swollen with purulent discharges.
C. Bright red bleeding and fluid collecting under the dressing with loss of pain control.
D. Bright red bleeding and fluid collecting under the dressing.
8. If catheter becomes disconnected from the tubing, the nurse should use which of the following
solutions to clean the tubing or connectors:
A. Alcohol
B. Povidone-iodine solution
C. Sterile water
D. Saline

9. The nurse instructs the client to report if she experiences signs and symptoms of local anesthetic
toxicity which includes the following:
1. Perioral numbness
2. Palpitations
3. Ringing in the ears
4. Seizures
A. 2 & 3
B. All of the options
C. 1, 2, 3
D. 3 & 4

10. The client describes the pain as knifelike chest pains that increase in intensity on respiration. Which
of the following systems is most likely its origin?
A. Pulmonary
B. Gastrointestinal
C. Cardiac
D. Musculoskeletal

Situation
Nurse Teresa assists in the care of a 25 year old male who is admitted in the emergency department for burns in
the chest, abdomen, right arm and right leg.

11.The physician orders total parentel nutrition (TPN) for the burn patient. Which of the following
statements is TRUE in this case? TPN is needed to _______.
a.Provide supplemental vitamins and minerals
b.Correct water and electrolyte imbalances
c.Ensure adequate caloric and protein intake
d.Allow the gastrointestinal tract to rest
12.Nurse Teresa is aware that fluid shifts occur during the emergent phase of a burn injury. This shifting
is due to fluid moving from what space? From ________.
a.Intracellular to extracellular space
b.Extracellular to intracellular space
c.Vascullar to intracellulr space
d.Interstitial to vascular space
13.Nurse Teresa understands that fluid shift results from an increase of which of the following?
a.Total volume of intravascular plasma
b.Total volume of circulating whole blood
c.Permeability of the kidney tubules
d.Permeability of capillary walls
14.Which of the following fluid and electrolyte imbalance would Nurse Teresa anticipate that the patient
would be particularly susceptible to in the emergent phase of burn care?
a. Hyperkalemia
b. Metabolic Alkalosis
c. Hemodilution
d. Hypernatremia
15.The patient is ordered to receive fluid resuscitation therapy. Nurse Teresa adjusts the infusion rate by
evaluating the patient’s:
a.Hourly body temperature
b.Hourly urine output
c.Hourly urine specific gravity
d.Daily body weight

Situation
Nurse Vance is a nurse manager of a surgical unit. He applies principles of resource management to improve client
care in the unit.
16. Nurse Vance supervises four nurses in the male surgical unit with forty clients in various stages of
care. Ten clients are receiving preoperative care; five are ambulatory and ready for discharge; ten are
scheduled for major operation; five are recovering from operation and ten clients are in rehabilitation.
Nurse Andy applies a concept in resource management which means making a choice among individuals
competing for scarce resources. This concept is ______
A. Rationing care
B. Technical scale application
C. Conflict resolution
D. Resource allocation

17. Nurse Vance applies an ESSENTIAL element in resource management in the surgical unit which is
A. Effective communication
B. Technical skills
C. Good interpersonal relations
D. Organization skills
18. Nurse Vance participated in budget preparation for the unit. Which of the following terms refer to
salaries of personnel, and cost of supplies and equipment?
A. Cost centers
B. Direct costs
C. Cost – benefit analysis
D. Indirect costs

19. Nurse Vance explains the concept of resource management to his staff. Which of the following
statements is TRUE about resource management?
A. Resource management is a principle in evidence based nursing practice.
B. Controlling the environment of the client.
C. Applying the management in client care.
D. Controlling the rising costs of health care while providing quality care of clients.
20. Nurse Vance participates in recruiting and retaining nursing staff and other personnel. This is an
example of what type of resources?
A. Financial resources
B. Human resources
C. Material resources
D. Nursing resources

Situation
A 51 year old male is admitted for complaints of rectal bleeding, abdominal pain, weight loss and change in bowel
habits. Nurse Ron is aware that these are manifestations of colon cancer. The physician prescribes radiation
therapy and chemotherapy.
21.Nurse Ron knows that radiation therapy is used to treat colon cancer before surgery. Which of the
following is the effect of radiation therapy? It ____.
a.Help heal the bowl after surgery
b.Eliminates malignant cells
c.Cures the cancer
d.Reduces the size of the tumor

22.The patient undergoes radiation therapy. Nurse Ron noted that the patient’s white blood cell (WBC)
count is severely depressed. The PRIORITY nursing intervention would be to ____:
a.Place the patient in a private room and maintain strict aseptic technique for all procedures.
b.Instruct the patient to avoid shaving with a sharp razor
c.Encourage visitors to visit the patient regularly to reduce the feelings of isolation the patient may feel.
d.Encourage the patient to include fresh fruits and green leafy vegetables in his diet.
23.Nurse Ron is aware that chemotherapy should only be administered by nurses who have taken special
courses in administering chemotherapy and who are highly skilled. Before the nurse gives the prescribed
dose of chemotherapeutic agent, the nurse should do which of the following:
a.Verify the dose, drug and schedule with another trained nurse.
b.Collect an extra syringe and needle in case of contamination.
c.Explain the expected side effects of the drug to the patient
d.Cover the patient with a water resistant shield
24.Nurse Ron assesses the patient receiving chemotherapy. Select the signs and symptoms that would
require further evaluation.
a.Patient complains of fatiuge
b.Hair loss on scalp
c.Large areas of ecchymosis in various sites on the body.
d.Dry mucous membrane

25.Nurse Ron writes a care plan for the patient receiving chemotherapy. Select what should be included
in the care plan.
1.Nursing Diagnosis 4. Interventions
2.Medical Diagnosis 5. Patient Education
3.Outcome 6. Evaluation
a.1,2,3,4,5
b.1,2,3,4,6
c. 1,3,4,5,6
d. 2,3,4,5,6

Situation
Nurse Marie, a charge nurse of an oncology unit attends a seminar on evidence based nursing practice.

26.The conference speaker explained the difference between evidence based pratice and evidence based
nursing practice. Which of the following is NOT TRUE regarding evidence based nursing practice?
Evidence- based nursing practice ____.
a.Has a medical focus
b.Uses a theory derived and research based information in making decision about nursing care delivery.
c.Considers the patient’s need and preferences based on nursing theory and research
d.Is a strategry used to improve patient care outcome.
27.The speaker encouraged the nurses to participate in the use of evidence- based nursing practice. The
speaker said that nurses must ____.
1.Participate in the development, use, or evaluation of practice guidelines.
2.Read and analyze outcome of research studies
3.Involve themselves in everyday patient care and nursing practice.
4.Know why they are doing what they are doing
a.2,3 and 4
b.1 and 2
c.1,2,3 and 4
d.1,3 and 4

28.Nurse Marie is aware of the importance of patient outcomes as a measure of quality care. Which of the
following statements is TRUE regarding patient-focused outcome indicators? To _______.
1.Achieve safe, quality, cost effective care for patients in daily practice.
2.Realize that individual nursing practice styles directly affect the rates at which patients recover.
3.Prevent development of unnecessary complications and injury.
4.Determine satisfaction rate from patients or family care received from nursing staff.
a.1,2,3 and 4
b.1 and 3
c.2 and 4
d.1,3 and 4

29.A model for using evidence- based practice was presented. The model has the following elements:
Plan, Do, Study and Act (PSDA). If nurse Marie wants to utilize this model to improve ward management,
what questions will she ask?

1.What are we trying to accomplish?

2.How well do we know that a change is an improvement?

3.Will the patient be satisfied with the improvement?

4.What change/changes can we make that will result in an improvement?


a.2 and 4
b.3 and 4
c.1 and 3
d.1,2 and 4
30.Nurse Marie is aware that patient care improvement must be based on which of the following?
a.Total cost of health care
b.Building and apply knowledge
c.Hospital policy on how to staff a nursing unit.
d.Accreditation standards

Situation
The hospice nurse cares for a 60-year-old female client with terminal illness.

31. To provide holistic care, the nurse assists the client in meeting her spiritual needs. Which of the
therapeutic nursing interventions BEST addresses the spiritual needs of the clients with terminal illness?
A. Use therapeutic touch
B. Communicate empathy
C. Play soft classical music
D. Pray with the client
32. As death approaches, the nurse provides invaluable support to the family. Other support people for
the family includes the ________
A. Funeral director
B. Spiritual adviser
C. Social worker
D. Physician

33. The client is experiencing dyspnea which is causing the client to be anxious. The nurse plans a holistic
approach of care for the client. the nurse should__________
A. Use an interdisciplinary approach
B. Allow the family to stay with the client
C. Prepare the client for a morphine injection
D. Secure an order from the physician liberal doses of anxiolytics
34. The hospice nurse assists the family to establish a relationship with the health care team by __________:
A. Supporting the decisions of the family
B. Allowing the family to take time to maintain good relation with the health care team
C. Explaining the roles of all interdisciplinary team members involved in the care of the client
D. Explaining to the family the client being cared of
35. The nurse assesses the client whose condition is deteriorating. A cardiovascular indicator of
imminent death is_____:
A. Narrowing pulse pressure
B. Bradycardia
C. Fluctuating blood pressure
D. Irregular heart rate

Situation
The nurse cares for a client with cancer who had lung surgery. The nurse educates the client on breathing exercise
and ambulation.

36.The nurse teaches the client how to deep breath effectively after a lobectomy. The nurse instructs the
client to:
a.Contract the abdominal muscles, take a deep breath through the mouth and exhale slowly as one trying to blow
out a candle.
b.Relax the abdominal muscle, take a slow deep breath through the nose, and hold it for 3 to 5 seconds.
c.Relax the abdominal muscle, take deep breath through the mouth and exhale slowly for 15 seconds
d.Contract the abdominal muscles, take a slow deep breath through the nose, and hold it for 3 to 5 seconds.
37.The client asks the nurse how much of his lungs are removed. The nurse responds based on
information that in lobectomy, a lobe is removed. In a wedge resection, which of the following is
removed?
a.A small, locatized are near the surface of the lung
b.One entire lung
c. Two lobes of the lung
d.A segment of the lung, including a bronchiole and its alveoli.

38.The client asks the nurse, what will fill the space where the lobe was? The correct response would be:
The _____.
a.Lung space will be filled up with serous fluid.
b.Surgeon filled the space with gel.
c.Space stays empty
d.Remaining lobe or lobes over expand to fill the space.
39.On the second post-operative day, the nurse auscultates the lungs and determines scattered crackles
bilaterally. Which of the following interventions would be MOST appropriate for the nurse to perform?
a.Encourage deep breathing and ambulation as soon as the client is able.
b.Encourage coughing and check the water seal system
c.Reduce the frequency of pain medications and increase the suction in the water seal bottle.
d.Perform endotracheal suctioning every shift.

40.The nurse teaches the client to perform which of the following exercises to prevent shoulder
ankylosis?
a.Raise and lower the head
b.Turn from side to side
c.Raise the arm on the affected side over the head
d.Flex and extend the elbow on the affected side

Situation
Nurse Faye admits a 30 year old female with tentative diagnosis of hypercalcemia.
41.Nurse Flora recognizes the signs and symptoms of hypercalcemia. Which of the following signs is an
indication of the diagnosis hypercalcemia?
a.Positive trousseau’s sign
b.Hyperactive bowel sounds
c.Hypertonicity of the muscles
d.Twitching

42.Nurse Faye recognizes the signs and symptoms of hypercalcemia. Which of the following signs is an
indication of te diagnosis hypercalcemia?
a.Muscle strength
b.Blood Pressure
c. Weight
d. Edema

43.The patient informs the nurse that she is taking Thiazide diuretics. Nurse Faye knows that Thiazide
diuretics drug are one of the most common causes of hypercalcemia. Which of the following signs should
the nurse observe for?
a.Increased peristalsis
b.Neurologic Depression
c.Neuromuscular Irritability
d.Decreased urine output

44.The patient goes into hypercalcemic crisis. Family members are anxious and worried. One relative
expresses to Nurse Faye “We don’t know what to do if she dies”. What is the BEST response of the nurse?
a.“Do not worry. We always see this kind of crisis and we can treat it.”
b.“I understand your concern, but I have to talk care of the patient first”.
c.“Yes, it is serious but I can come back, talk to you and answer your questions.”
d.“Has your loved one been eating processed food and drinking alcoholoic beverages lately?”
45.Fortunately, the patient recovers from the hypercalcemic crisis. The patient is ready for discharge.
Which of the following activities should be included in the discharge plan? Instruct the patient to ____.
a.Take in anti-diarrheal medications as prescribed by the physician
b.Encourage foods that increase urine acidity
c.Decrease sodium and calcium intake
d.Restrict fluid intake to less than 1 liter a day
Situation
A nine year old male child is hospitalized for burns on the right arm, right leg and abdomen. The nurse documents
the treatment performed on the child.
46.The nurse determines the extent of burns using the rule of nines. Which of the following assessment
findings should the nurse document?
a.18% of the child’s body surface is burned.
b.45% of the child’s body surface is burned.
c.50% of the child’s body surface is burned.
d.25% of the child’s body surface is burned.

47.The nurse writes a nursing diagnosis for the child which is the basis of care for the first 24 hours of
admission. The MOST appropriate nursing diagnosis would be _______:
a.Fear and Anxiety
b.Disturned Body Image
c.Risk for Infection
d.Impaired Mobility

48.The physician writes an order for the client. Infuse D5 water 500 cc to run for 8 hours. The IV micro
set delivers 60 drops per ml. How many drops should the nurse regulate the flow and record it in the
client’s chart?
a.50 drops per minute
b.62 drops per minute
c.35 drops per minute
d.30 drops per minute
49.the nurse commits an error in documenting the care of the burnt child. She consults the charge nurse
to find out if the hospital has an established policy on correcting documentation errors. Which of the
following is an accepted form for correcting errors?
a.Enclose in parenthesis the erroneous statements, draw a line across the statement, and make the correct entry
above the 1 drawn.
b.Correct the error by applying correction fluid or tape and write the correct entry over it.
c.Cross through the erroneous word or statement with a double line, affix your initials, write the pharse “mistaken
entry” then write the correct information.
d.Use the slide rule method. Cross through the erroneous word or statement with a single line, affix your initials,
write the date and time the correction was made, the write the correct information.

50.The nurse is aware that documentation requires the following EXCEPT: Documentation should:
a.Be systematic and organized
b.Comply with policy standards of the health care facility.
c.Present exact and correct details pertinent to the event.
d.Include reactions and interpretations of the nurse on the event.

Situation
Ms. Sunshine, an oncology nurse assists in the care of a 49 year old female client diagnosed with breast cancer.
51. The client asks Ms. Sunshine what part of the breast is cancer usually found. Based on her knowledge,
the nurse says that most malignant tumors occur in the ____ of her breast.
A.Lower, inner quadrant
B.Upper, inner quadrant
C.Lower, outer quadrant
D.Upper,outer quadrant
52. The physician orders a modified radical mastectomy right. The client returns to her room after the
surgical procedure. Which of the following positions will the nurse place the right arm of the client?
A.Across the client’s chest wall
B.In a position that affords the client’s greatest comfort without placing pressure on the incision
C.At her side at the same level of her body
D.Elevate on pillows with client’s hand higher than her elbow and her elbow higher than her shoulder

53. The client has radiation therapy after a modified radical mastectomy. In caring for the skin at the site
of the therapy, the nurse instructs the client to avoid all of the following practices EXCEPT to:
A.Wash the area the water
B.Apply an ointment to the are
C.Use talcum powder on the area
D.Expose the area to sunlight
54. The nurse further explains to the client that a normal local tissue response to radiation is:
A.Scattered pustule formation
B.Atrophy of the skin
C.Slouching of two layers of skin.
D.Redness of the surface tissue
55. Ms. Sunshine understands that the risk factors for the development of breast cancer include the
following EXCEPT:
A.Early onset of menstruation
B.Family history of breast cancer
C.Breast feeding
D.Delayed onset of menopause

Situation
The nurse assists in the care of female clients. KC is a 35 year old woman with hyponatremia. According to the
client she is taking diuretic medications.

56. Which of the following statements is TRUE about hyponatremia?


A. Hyponatremia from diuretic use may produce small quantities of urine.
B. Hyponatremia occurs because of excess fluid volume diluting the potassium.
C. A serum sodium level determined above 135 mEq/L indicates hyponatremia.
D. Hyponatremia from diuretic use may produce large quantities of urine.
57. In assessing the client, the nurse should focus on which part of the following? The ______:
A. Spiritual state of the client
B. Physical signs and symptoms
C. Diagnostic to be done on the client
D. Mental status of the client
58. The nurse writes a nursing diagnosis. Which of the following is appropriate?
A. Disturbed thought processes
B. Decreased cardiac output
C. Activity intolerance
D. Ineffective breathing pattern

59. The client has a serum sodium level of 115 mEq/L. a priority nursing intervention is for the nurse to:
A. Give frequent oral care
B. Take precautions for occurrence of seizures
C. Monitor cardiac rhythm
D. Take the vital signs every two hours
60. The nurse is much aware that a client receiving D5W at 100 ml/hr. is MOST at risk for developing
which of the following conditions?
A. Hyponatremia
B. Fluid volume excess
C. Hypernatremia
D. Fluid volume deficit

Situation
Nurse Liza admits a 25 year old female for severe angioedema involving the face, hands and feet. The patient
further complains of burning and stinging of the lesions.
61.The nurse interviews the client. Which of the factors would be a MOST significant risk factor for
allergies?
a.A recent upper respiratory infection
b.Family history of allergic reactions
c.Living in a third world country
d.Exposure to fungal infection

62.Based on assessment findings, nurse Liza writes a nursing diagnosis is:


a.Risk for Injury
b.Altered Comfort
c.Risk for Infection
d.Impaired skin integrity
63.The patient undergoes allergen testing using the cutaneous scratch method. To prevent anaphylaxis,
nurse Liza should initially monitor the patient’s ____.
a.Bilateral lung sounds
b.Arm at the site of the skin testing
c.Pupil’s size and reaction to ligh
d.Blood pressure and pulse

64.Nurse Liza knows that allergic reactions usually occur within minutes after injection of an allergen.
The nurse should ____.
a.Administer high flow oxygen
b.Document the patient’s allergy history
c.Observe the patient for about 20 minutes after injection
d.Monitor the patient’s bloos pressure and pulse
65.The physician orders Epinephrine 1:10,000 0.5ml IV. After administering the drug, the NEXT action of
nurse Liza is to ___.
a.Start oxygen at 100% using a non-breather mask.
b.Prepare an infusion of dopamine (Intropin)
c.Administer diphehydramine (Benadryl) IV.
d.Give a dose of cimetidine (Tagamet).

Situation
The nurse attends a session on alternative and complementary for pain management. Some of the information she
acquires are discussed in the following situations.
66. The nurse learned in the session that complementary therapies for pain management ____________:
A. Will work only if the patient and the therapist believe it will work.
B. Should be prescribed when opioids are no longer effective.
C. Have no place in moderate to severe pain management.
D. Maybe used at any time as adjunct therapy.
67. Time travel is a _____:
A. Form of imagery where the patient visualizes a time when his/her pain is relieved.
B. Form of hypnosis that maybe useful in a number of conditions.
C. Stress reducer where the client visualizes himself or herself in a quiet environment.
D. Form of distraction in which the person places himself in an imaginary environment.
68. Applying pressure to which of the following organs would be considered reflexology that would
eliminate specific diseases or conditions. The areas would be the ____:
A. Neck and face
B. Abdomen and back
C. Feet, hands, and ears
D. Eyebrows and forehead

69. An agent that is used in aromatherapy to decrease anxiety that often accompanies pain is ____:
A. Ginger
B. Lemon grass
C. Black pepper oil
D. Lavender
70. An agent that is used in aromatherapy to decongest and reduce discomfort associated with cold
allergies is ______:
A. Lemon grass
B. Eucalyptus
C. Lavender
D. Black pepper oil

Situation
The nurse assists in the care of a ten-year-old male child with bronchial asthma. The nurse documents care given
to the child.
71. The nurse completes an admission assessment on the child primary to ____:
A.Diagnose if the client is at risk for falls.
B. Identify clusters of data.
C. Establish a therapeutic relationship.
D. Ensure that the bronchial asthma attacks are controlled.

72. On admission, the nurse records the following data. According to the mother, the child has a history of
asthma attacks triggered by exposure to cold, smoke and nuts. Vital signs taken and recorded as follow:
temperature – 99.4 ⁰F; RR – 36 breaths per minute; PR is 160 beats per minute. This is an example of
what form of charting?
A. Narrative charting
B. SOAPIE charting
C. Focus charting
D.Problem–focused charting

73. Which would be an example of objective data that the nurse records?
A. Vital signs are: T = 99.4 ⁰F; PR = 160 b/min; RR = 36 b/min
B. Patient has difficulty of breathing
C. Patient feels tired
D. Patient feels cold.
74. Once the data is collected, the first thing the nurse does is to ______:
A. Determine the significance of the information.
B. Formulate a nursing diagnosis.
C. Write a client-centered goal.
D. Design a plan of nursing interventions.
75. The nurse reads the chart of the patient. What part in the chart would the nurse find documentation
of the current medical diagnosis?
A. Client’s flow sheet
B. Admission sheet
C. Nurse’s notes
D. Physician’s progress notes

Situation
A landslide had occurred in x province due to persistent and heavy rainfall. According to information received by
the Emergency Department Staff, there were about 150 people injured.
76.The nurse prepares the department to receive the injured. This situation would be BEST classified as
a/an:
a.Unnatural Calamity
b.Natural Calamity
c.Mass Casualty Incident
d.Accidental Disaster

77.The first responders to the scene of disaster would MOST likely use which of the following tools to
separate victims for easy recognition of those in need of immediate care.
a.Number Codes
b.Triage Tag
c.Category Tags
d.SALT Triage
78.The ED nurse plans the coding for disaster victims. Which colors are MOST associated with triage?
1.Green
2.Yellow
3.Black
4.Blue
5.White
a.2,3,4,5
b.1,2,3,4,5
c.1,2,3,4
d.1,2,3
79.The ED nurse receives a victim with severe life- threatening injuries and most likely wil not survive.
The victim would be triaged as:
a.Yellow
b.Red
c.Black
d.White
80.One of the victims, a female was brought to the ED crying hysterically and looking frightened. She
exclaims she was inside her house when the landslide occurred and just barely escaped. What is the BEST
action by the nurse?
a.Refer her to the proper authorities for counselling.
b.Triage her and give appropriate treatment.
c.Call a physician to talk to her.
d.Advise her to go home with a relative.
Situation.
A 45-year old male is admitted for a productive cough with thick yellow sputum and shortness of breath which
according to him has increasingly progressed for the past four weeks.
81. Based on the information gathered, the nurse formulates a nursing diagnosis appropriate for the
patient which is ____:
A.Self-Care Deficit related to fatigue secondary to increased work of breathing and insufficient ventilation.
B.Ineffective Breathing Pattern related to shortness of breath and productive cough.
C.Activity intolerance related to fatigue.
D.Ineffective Coping related to anxiety and depression.

82. The nurse writes the nursing diagnosis which is the basis of care for the patient. Which of the
following is an expected outcome for the patient?
A.Improve breathing pattern
B.Achieve airway clearance.
C.Verbalize feelings
D.Improve activity tolerance.

83. The nurse reads an article on risk factors for COPD which include environmental exposures and host
factors. Which of the following is the MOST important environmental risk factor worldwide?
A.Cigarette smoking
B.Passive smoking
C.Getting order in age
D.Exposure to occupational chemicals and dust.
84. The nurse performs an assessment on the patient. Which of the following actions is a priority by the
nurse.
A.Refer the patient immediately to the physician
B.Obtain a thorough health history.
C.Schedule the patient for spirometry
D.Assess level of pain

85. The nurse recalls reading a research article on “Burden of Lung Disease”. The findings revealed that
“women in rural areas are also vulnerable to lung disease from inhaling ethanol or gases from the
charcoal and wood fire”. This study is described as _____:
A.Retrospective
B.Qualitative
C.Quantitative
D.Historical

Situation
The director for research and development assigns Nurse Gelli to be the principal investigator of a research project
being undertaken to improve nursing services. The following questions relate to ethics in research.

86. Nurse Gelli understands that the subject in any study must be guaranteed of all the following EXCEPT:
A.Financial compensation
B.Privacy
C.Confidentiality
D.Protection from harm
87. For an informed consent to be ethical, Nurse Gelli must _____:
A.Provide a clear explanation of the research project.
B.Contact a lawyer.
C.Make sure a third person is present to witness the signing of the informed consent.
D.Make a recording of the consent
88. Nurse Gelli understands that revealing names of subjects in a published research report is _________:
A.A popular move
B.A normal procedure
C.Ethical
D.Unethical
89. Nurse Gelli prepares a set of questionnaires. She places an identification number on the corners of the
questionnaires corresponding to the master list of names and numbers of the subjects. This is to assure
the subjects that the information they provide will not be shared with anyone. This principle is called
_________:
A.Anonymity
B.Privacy
C.Confidentiality
D.Data security

90. One of the subjects in the ongoing study wants to withdraw from participating. Nurse Gelli should
_______:
A.Permit the subject to withdraw without prejudice
B.Replace the subject with another subject
C.Talk with the subject about his or her reason for withdrawing
D.Complete the study with one less subject

Situation
Nurse Kath assists in the care of female patients with coronary artery disease (CAD). She schedules time to educate
these groups of women about CAD.

91. A correct statement about CAD in women is that _____:


A.Hormone Replacement Therapy is recommended for prevention of coronary artery disease.
B.Women develop CAD earlier than men.
C.The genetic component for CAD is weak.
D.The rate of women having CAD is steadily rising while it is declining in men.
92. Research indicates that a woman with CAD needs to exercise to decrease the risk of having CAD.
Which of the following exercises is recommended?
A.Light to moderate exercise for 30 minutes 5x a week.
B.Light exercises (walking) 20 minutes 3x a week.
C.Aggressive exercise for 30 minutes 3x a week
D.Moderate exercise for 20 minutes 5x a week.

93. Nurse Kath gives information about blood pressure in women. Which of the following statements is
correct?
A.Hypertension doesn’t affect CAD risk as women age.
B.Low blood pressure is twice as common as oral contraceptive users.
C.Twenty percent of women have hypertension before menopause.
D.Weight, age, and oral contraceptive use affect blood pressure.
94. Nurse Frances explains that stress can be managed by which of the following:
A.An individual has low and constant stress
B.An individual has high stress level and low control
C.An individual has high control and low stress level
D.Stress is controlled over short periods.

95. Nurse Frances explains that stress can be managed by which of the following:
A.Socializing with other patients with similar disease
B.Taking in prescribed medications to relieve you of stress.
C.Finding spiritual meaning in what you are experiencing
D.Reflecting on your condition and accepting it.
Situation
Nurse Pau specialized in emergency nursing. She collaborates with ED physician, triage team and other members
of the health team provide care to clients of all afe- groups with various illnesses or injuries coming to the
emergency department.

96.When clients are wheeled into the ED, Nurse Pau applies a triage priority rating system. When a client
must be treated immediately otherwise the client’s life, limb or vision are threatened, the category is ____:
a.Low urgent
b.Urgent
c.Emergent
d.Non Urgent
97.A client with spinal cord injury due to trauma is brought to the ED by EMS personnel. Nurse Pau
assesses the client. Which of the following will the nurse suspect the client to manifest?
a.Tachycardia
b.Diaphoresis
c.Pain
d.Temporary Loss of relax function
98.The neurosurgeon performs a neurological assessment on the client. Based on the Glasgow Come
Scale, the client has a total score of 8. Nurse Pau recognizes this score to indicate which type of injury?
a.Severe injury
b.Minor Injury
c.Moderate injury
d.No injury

99.Nurse Pau writes a nursing diagnosis. Which of the following diagnoses is a PRIOPRITY nursing
diagnosis for a client with spinal cord injury?
a.Risk for infection
b.Dysreflexia
c.Ineffective airway clearance
d.Ineffective breathing pattern
100.The neurosurgeon determines the client’s spinal cord injury is at the level of T5. Nurse Pau is alerted
when the client complains of severe headache, is diaphoretic, a head and neck appear to be flushed. She
takes the vital signs. Plus rate is 47 beats per mintue and blood pressure is 220/114 mmhg. Based on the
assessment data, nurse Pau concludes that the client need IMMEDIATE treatment for _____:
a.Spinal shock
b.Autonomic Dysreflexia
c.Pulmonary Embolism
d.Malignant Hypertension
Situation
You are a staff nurse in a Rehabilitation Center for Substance Abuse.

1. You admitted an intoxicated patient for alcohol withdrawal. Which of the following interventions
should you implement to help the client become sober?
A. have the patient take a cold shower
B. walk the patient around the unit
C. Provide the patient a quiet room to sleep in
D. Give patient a black coffee to drink
2. While obtaining the history of a patient with several cases of driving under the influence of alcohol, you
asked about the amount of alcohol the patient consumes daily. He answered “I just have a few drinks with
the guys after work”. Which of the following would be your MOST therapeutic response?
A. “That’s all the patients here say at first”.
B. “You say you have a few drinks, but you have a multiple arrests”.
C. “I think you cannot handle a few drinks”.
D. “Then you should have somebody driving for you”.
3.Which of the following assessment data provides the best information on the patient’s physiologic
response and the effectiveness of the medication prescribed for alcohol withdrawal?
A. Sleep pattern
B. Evidence of tremors
C. Vital signs
D. Nutritional status

4. One of your patients is manifesting signs and symptoms of alcohol withdrawal such as: tremors,
diaphoresis, and hyperactivity. Blood pressure is 190/92 mmHg and pulse rate of 92 beats/min. which of
the following medications should you expect to be ordered for this patient?
A. Lorazepam (Ativan)
B. Naloxone (Narcan)
C. Haloperidol (Haldol)
D. Benztropin(Cogentin)

5.A patient discharged from an alcohol rehabilitation program was on Clonazepam (Klonopin) 0.5 mg.
three times a day. Several months later he reported having insomnia, shakiness, sweating, and one
seizure. Which of the following questions should you ask FIRST? Ask if he _____________.
A. has stopped taking the Klonopin suddenly
B. has been drinking alcohol with the Klonopin
C. has developed tolerance to the Klonopin and needs to increase the dose.
D. is having a panic attack and needs to take extra Klonopin.

Situation
You are a staff nurse in the Neurologic Unit of the Hospital. You are taking care of patients with different
neurological conditions. The following questions refer to this statement.

6. Patients suffering from Spinal Cord Injury are at risk for hypotension. This risk is attributable to ______.
A.Volume depletion
B. Pulmonary overload
C.Loss of reflexes
D.Fluid depletion

7. Which of the following can be used as a guide when establishing the functional ability of a patient with
Spinal Cord Injury?
A.Degree of Residual Muscle Function
B.Absence of Hypotension
C.Degree of Independence
D.Absence of Complications
8. Benjamin, a 28 year old stuntman, was admitted to your unit unconscious, after falling from a horse.
Tentative diagnosis of Spinal Cord Injury was the cause of?
A.Motor Vehicle Crashes
B.Sports
C.Violence
D.Falls

9. The final diagnosis for Benjamin is Spinal Cord Injury, Anterior Cord Syndrome. Which of the following
is NOT a characteristic of this condition?
A.Loss of motor power
B.Loss of pain and temperature sensations
C.Pronounced sensory loss in the upper extremities
D.Preserved position, vibration and touch sense
10. Patients with Spinal Cord Injury benefit from rehabilitation. Which of the following is NOT a goal of
rehabilitation?
A.To promote independence
B.To prevent infection
C. To reduce morbidity
D.To maximize function recovery

Situation
One of your programs as a community health nurse is related to substance abuse addiction. You spearheaded an
educational collaborative activity to address the increasing drug and alcohol problems in the community. A multi-
sectoral committee was formed, and an initial organizational committee meeting was held. The following questions
apply to this situation.
11.To motivate and spark the interest of prospective participants in the proposed program, which of the
following activities will you introduce as an initial activity of the group?
A.Lecture-forum on alarming dangers of drug and alcohol use.
B.A summer sport fest involving all age group.
C.Values-formation program
D.Lecture-forum to increase awareness of unhealthy lifestyle.
12.To assess the prevalence and incidence of substance abuse, a community survey was done by the
barangay health workers. Using purposive sampling, what age group is at MOST risk to be considered as
the survey respondents?
A.Pre-schoolers
B.Older adults
C. Older Children
D. Adolescents

13.A self-help support group for educational and caring approach for alcoholics is:
A.Halfway homes
B.Psychiatric treatment and rehabilitation center
C.Alcoholic Anonymous
D.Detoxification treatment programs
14.The LEAST effective education and preventive measure for drug abuse is:
A.Activity workshops for out of school youths
B.Family enrichment programs
C.Regular advocacy meetings to include families, store, restaurant, bar owners
D.Leaflets and brochures for mass distribution
15.The committee organized parent education classes. A representative from the local police department
was invited to discuss:
A.Stress management
B.Parent effectiveness training
C.The legal implications of illicit drug use
D.Recognition of deviant child and adolescent behavior

Situation
The Nursing Service Department of the hospital you are working in has planned a training program on
Computerized Documentation. To keep yourself updated with advanced technology, you signed up for it.
16. What equipment is NOT considered technologically sophisticated?
A.Blood gas analyzer
B.Patient-controlled analgesia
C.Computerized infusion pumps
D.Chest X-ray machine

17. Which of the following is a language system that includes common nursing terms from a variety of
vocabularies?
A.Cochrane Collaboration System
B.National Information Center
C.Unified Nursing Language System
D.NANDA
18. The “virtual office visit” is a growing trend in patient care. Which of the following is NOT TRUE of this
type of visit?
A.The nurse communicates with the patient via audio monitors
B.The patient and the nurse are able to view and speak to one another
C.The patient and the nurse are able to exchange information
D.It uses e-mail

19. The specialty in which nurses are considered experts in healthcare information technology is _______.
A.Information Specialist
B.Nursing Informatics
C.Computer System Expert
D.Advanced Practice Nursing
20. Which of the following chronic disease conditions is NOT CONSIDERED in tele-monitoring?
A.Chronic renal failure
C.Congestive Heart Failure
B.COPD
D.Multiple Sclerosis
Situation
Cecile is a beginning nurse in a government hospital. She still needs enhancement of her competencies in charting.
21.The following are methods of charting, EXCEPT __________.
A.Sequential
B.Narrative
C.POMR
D.Focus
22.In a PIE method of charting, P is for needs, EXCEPT ___________.
A.Admission
B.Problem
C.Teaching
D.Discharge
23Which statement is NOT true about medical record that Cecil must understand. Charts _____________.
A.Contents must be kept confidential
B.Can be borrowed by any nursing student
C.Can be accessed by a researcher.
D.Can be given out only with patient’s consent
24 Cecil should know that in narrative charting, documentation of patient care should be ____________.
A.Extensive
B.Formatted
C.Chronologic
D.Descriptive

25. In charting neurologic assessment, Cecil must understand that the following should be indicated.
SELECT all that apply.
1.Orientation 3. Sensation
2.Pupil movement 4. Quality of speech
A.3 and 4
B.1 and 2
C.1, 2, 3, and 4
D.1, 3, and 4

Situation
You are a staff nurse in the Eye Unit of the hospital. In the regular Quality Circle meeting one of the matters
discussed was the increasing complaints of patients who underwent eye surgery in relation to instillation of eye
drops. The case of Julia who has cataract surgery was presented as a frame of reference in the reviewing the
procedure on eye drops instillation.
26.In instilling the eye drops you have to tilt the back slightly Julia’s head. Which of the following is the
rationale for this nursing action?
A. to prevent any spill from the eye during the procedure
B. to keep any debris away from the lacrimal duct.
C. to facilitate the procedure.
D. to reach the conjunctival sac easily.
27.You have observed presence of drainage in Julia’s eyes. Which of the following eye solutions would
you use in cleaning her eyes?
A. hydrochloric acid
B. Normal saline solution
C. Mineral water
D. Soapsuds solutions

28.You cannot open Julia’s eyes due to dried crust. Which of the following actions would you do?
A. Apply the eye drops the next day
B. Apply eye ointment to soften the crust.
C. Instill eye drops
D. Place a warm wet wash cloth over her eyes for at least three minutes.

29.You accidentally touch Julia’s eyelid during eye drop instillation causing her to blink. What should
have you done to prevent this from occurring?
A. Touch the inner side of the medicine cap
B. Tilt back Claudia’s head slightly
C. Hold the dropper close to the eye but do not allow the medication to fall into the cornea.
D. Have Claudia look up and focus on something
30.Which of the following is the rationale for instilling eye drops to patients who has eye surgery?
A. to help control the intra-ocular pressure.
B. to dilate the patient’s pupils
C. to treat an eye infection
D. to constrict the patient’s pupils

Situation
You are a staff nurse assigned at the Musculo-Skeletual unit of the Pediatrics Department.
31. You do the initial assessment of a child diagnosed to have OSTEOMYELITIS, LEFT TIBIA. Which of the
following would you expect to find when assessing the area over the tibia?
A.Diffused tenderness
B.Decreased pain
C.Localized edema
D.Increased warmth
32. Which of the following should you include in the care plan to meet the development needs of an 8
year old child confined at home with osteomyelitis?
A.Talking to the child about his interests twice daily
B.Allowing siblings to visit throughout the day
C.Encourage the child to communicate with school mates
D.Encourage the parents to stay with the child
33. A child with newly diagnosed osteomyelitis has nausea and vomiting. The parents wanted to give
ginger candy to help control the nausea. Which of the following should you tell the parents?
A.“Your child needs medication for the vomiting”
B.“I will need to get a prescription”
C.“We discourage the use of home remedies in children”
D.“You can try them and see how it does”
34. A child is to receive IV antibiotics for osteomyelitis. Which of the following blood tests should be done
before the initial dose of antibiotics can be given?
A.Culture
B.White blood count
C.Creatinine
D.Hemoglobin

35. You are caring for a child with osteomyelitis who will be receiving high-dose intravenous antibiotic
therapy for 3 to 4 weeks. Which of the following should you plan to monitor?
A.Urine glucose level
B.Thrombin time
C.Blood glucose level
D.Urine specific gravity
Situation
Nurse Jenny who is on-duty in a psychiatric unit is assigned to care for patients at-risk for suicide.
36.In evaluating the effectiveness of the care provided for a self-destructive patient, the BEST approach is
to __________.
A. modify the plan as little as possible to avoid confusing the patient.
B. make sure the staff has followed the original care plan.
C. involve the patient in the process of evaluation.
D. identify maladaptive coping behaviors.
37.Which of the following statements would BEST represent Nurse Jenny’s attempt to assess a patient’s
current ability to organize and enact a suicide wish?
A. “What is your educational background?”
B. “What plan do you have for committing suicide?”
C. “Have you ever thought about hurting yourself?”
D. “Are your self-destructive thoughts frequent?”

38. Nurse Jenny should pursue assessment of suicide risk for individuals who display tendencies to be ___.
A. Compulsive, obsessive, or weak
B. risk-taking, aggressive, or controlling
C. hostile, impulsive, or depressed
D. blaming, abusive, or confused

39.On admission, the nursing diagnosis for Harry, who is depressed and suicidal is, “Risk for suicide.” An
appropriate outcome for this diagnosis at discharge from the hospital is, “The patient will _____________.
A. not harm self while hospitalized
B. be able to problem-solve effectively
C. increase feelings of self-worth
D. develop a trusting relationship with the nursing staff.
40.A male suicidal patient is found by Nurse Jenny as he tries to hang himself in the bathroom. What
nursing intervention would address the patient’s need for safety while maintaining his self-esteem?
A. Advise him to use the bathroom only with staff supervision.
B. Tell him that the police is coming to rescue him.
C. Place him in the seclusion room with 15- minute checks.
D. Assign a nursing staff to remain with him all times.

Situation
As a beginning professional nurse one of the competencies expected of you is to engage in research. Basic
knowledge in research, therefore, is very important. The following questions related to these statements.

41.Which of the following is a type of probability sampling?


A.Cluster
B.Purposive
C. Quota
D. Convenience
42.You want all people who are currently institutionalized for psychiatric problems to participate in your
study. This is an example of ____________.
A.Universal population
B.Target population
C.Element
D.Sample

43.What element distinguishes a true experimental research from a quasi-experimental research?


A.The use of non-probability sampling
B.Lack of random assignment of subject to group
C.Size of the sample
D.The introduction of experimental treatment.

44.You want to determine the difference of the anxiety level between male and female patients scheduled
for abdominal surgery. The first 30 males and the first 30 females who are admitted to the hospital at the
time of the study would be asked as study participants. This is an example of what sampling technique?
A.Fish bowl technique
B.Simple random
C.Quota sampling
D.Purposive sampling
45.You want to determine the effect of a planned exercise on low-back pain. Which of the following
research designs would be MOST APPROPRIATE to use in this study?
A.Posttest only control group design
B.One-shot case study
C.Pretest-posttest control group design
D.One group pretest-posttest design

Situation
You are a staff nurse in the Psychiatric Unit of the hospital taking care of patients with manic disorder. The
following questions relate to this statement.
46.What would be your MOST therapeutic response to a manic patient demanding that you call his
attending physician to make an order for a pass to go out at the middle of the night?
A. “You go to the recreation hall now while I call your attending physician”
B. “You must really be upset to want a pass immediately. I will give you a medication to make you calm.”
C. “I can’t call your physician now but you can talk to me about your request to pass.”
D. “Don’t be unreasonable. I can’t call your attending physician in the middle of the night”

47.One of your patients lacks food and fluid intake due to poor appetite. What foods would BEST meet the
patient’s nutritional needs?
A. Steak and spaghetti
B. Peanut butter, sandwich and milk
C. Carrots, celery, raisins, apple
D. Beef and non-diet soda
48.During lunchtime you have observed that a patient with mania taking food from other patient. What
principle would guide you in determining the appropriate intervention in this situation?
A. the patient needs foods and fluids any way possible
B. The patient will calm down as soon as lunchtime is over.
C. the intrusive behavior of a manic patient is not a threat to other patients.
D. other patients need to be protected from the intrusive behavior of other patients.
49.A patient is manifesting some manic depression. Which of the following activities would you provide
to distract such patient?
A. you let the patient go for a walk
B. give him a book to read.
C. let him watch the final game of the UAAP
D. let him play checkers with another patient.

50. What therapeutic activity would you provide a patient with mania?
A. doing ceramics and whittling
B. playing cards and checkers
C. playing bingo and knitting
D. drawing and folding towels

Situation.
You are a staff nurse in the psychiatric unit of the hospital. One of the most common conditions you care for are
patients with anxiety. The following questions relate to this statement.

51. The patient with obsessive-compulsive disorder attempts to control anxiety through ritualistic
behaviors. Which of the following nursing interventions will increase the patient’s sense of security?
A.Distract the patient from rituals with other activities
B.Stop the client from doing the rituals
C.Encourage the patient to talk about the purpose of the rituals
D.Allow the patient to perform the rituals
52. You are teaching a patient with generalized anxiety about foods. What would you instruct the patient
as to what food to avoid?
A.High-fat foods
B.Caffeine
C.Sodium
D.Refined sugars

53. Distorted perceptions, loss of rational thinking, and failure to perceive potential harm to
characteristics of which level of anxiety?
A.Severe
B.Panic
C.Mild
D.Moderate
54. A patient with panic disorder is learning to counter negative thoughts of “I’m going to die” with “This
is only anxiety and it will pass”. The patient is using which of the following THERAPEUTIC APPROACHES?
A.Cognitive restructuring
B.Behavior modification
C.Systematic desensitization
D.Conscious relaxation
55. You have a patient having a panic attack. Your MOST APPROPRIATE action is to ______.
A.Stay with the patient while allowing an adequate personal space
B.Explain in detail what is happening to promote the patient’s understanding
C.Place the patient in an area with other people who can offer support
D.Reassure the patient that the situation is not as bad as he thinks it is

Situation
You are the Quality Assurance Officer of your hospital. The following questions relate to this statement.
56. As the Quality Assurance Officer, you developed a group of nurses to conscientiously use the current
evidence in managing patients with rheumatoid arthritis. This is an example of _______.
A.Evidence-based practice
B.Purposive sampling
C.Research utilization
D.Random sampling

57. Based on the initial research findings, Sterillium, an alcohol-based hand disinfectant is more effective
than the traditional brush-stroke hand scrub method. You recommend its use by the nurses in the OR.
This is an example of ______.
A.Generalizability of results
B.Evidence-based practice
C.Dissemination of research findings
D.Research utilization

58. One of your programs as a Quality Assurance Officer is to develop the research culture in the Nursing
Service Department. What would be the best and easiest thing to do to encourage the nursing personnel
to participate in research?
A.Teaching them statistical procedures commonly applied in research
B.Letting them do a critical analysis of a research topic
C.Doing a review of related literature and studies
D.Asking them to participate in gathering data
59. You were asked to present the most frequently occurring causes of sentinel events for the last six
months as the basis for the patient safety program. What measure of central tendency are you going to
use in this situation?
A.Mode
B.Median
C.Mean
D.Analysis of variance

60. You want to determine the difference in the coping skills among adolescents diagnosed with drug
abuse as a basis of a gender specific program on coping skills. What will be your sampling technique to
answer the research problem?
A.Stratified sampling
B.Purposive sampling
C.Convenience sampling
D.Random sampling

Situation
Mrs. Sam Reyes, a 38 -year old female came in for consult with the chief complaint of severe pain of both eyes. She
claimed she sometimes sees rainbow halos around light. She also experiences nausea and vomiting, the attending
physician gave an impression of Acute Angle-Closure Glaucoma.
61.Vision loss for Mrs. Reyes may progress as days go by and this is irreparable. An appropriate nursing
diagnosis would be ___________.
A. disturbed sensory perception related to severe pain
B. risk for ineffective therapeutic regimen
C. disturbed sensory perception related to recent loss of vision.
D. grieving related to loss of vision

62.In a medically-managed patient with glaucoma, independent self-care is the area for evaluation. Which
of the following is a short-term outcome for Mrs. reyes?
A. ability to recognize clinical manifestations of complications
B. Mrs. Santos’ self-care ability
C. Compliance with the medical management
D. Mrs. Reyes ability to instill eye drops
63. Mrs. Reyes, what demographic data is/are RELEVANT to ocular movement?
A. gender
B. age at menarche
C. age and gender
D. Race
64.An opthalmoscopic examination was done to Mrs. Reyes. The following results are expected EXCEPT
____________.
A. nonreactive pupil
B. corneal cloudiness
C. erythematous conjunctiva
D. decreased intraocular pressure
65.Select the medication that should be discussed by the nurse with the attending physician before
administering it to Mrs. Reyes.
A. Isoptocarpine eyedrops
B. Corticosporin ophthalmic
C. Mannitol
D. Atropine
Situation
You are a staff nurse in the psychiatric unit of the hospital taking care of patients with various psychiatric
disorders. The following questions relate to this statement.
66. You are concerned about the lack of food and fluid intake of one of your patients with mania. What
foods should you select that would BEST meet the patient’s nutritional needs?
A.Peanut butter, sandwich and milk
B.Carrots, celery, raisins, apple
C.Beef and non-diet soda
D.Steak and baked potato

67. Which of the following activities would you select to provide distraction for a patient with manic
behavior?
A.Reading a book
B.Plating checkers
C.Going for a walk
D.Ceramics and whittling

68. Which of the following would include therapeutic activities for the patient with mania?
A.Bingo and knitting
B.Playing Cards and checkers
C.Drawing and folding towels
D.Ceramics and whittling

69. You observed a patient with mania taking food from another patient’s food tray during lunch time.
Your nursing intervention should be based on what rationale?
A.The patient’s behavior is not an important threat to anyone’s physical safety
B.The patient needs food and fluids in any way as possible
C.As soon as lunch is over, the patient will calm down
D.Other patient’s need to be protected from the intrusive behavior of other patients.
70. Few minutes past midnight, a patient with mania approached you in the nurse’s station. He demands
that his psychiatrist come to the unit now and write an order for a pass to go home. Your MOST
THERAPEUTIC response is ________.
A.“I can’t call your psychiatrist now but you can talk to me about your request for a pass”
B.“You must really be upset to want a pass immediately. I will give you PRN medication.”
C.“Go to the recreation room while I call your psychiatrist”
D.“Don’t be unreasonable. I can’t call your psychiatrist at this time of the night.”

Situation
You are a staff nurse in the Eye Unit of the hospital. You want to find out the evidence-based care for the nurses to
improve adherence to eye drop therapy. You formulated the clinical question “Among patients with Glaucoma
which nursing interventions increase adherence to eye drop treatment?”
71. Which of the following is the expected outcome of the clinical question?
A.No expected outcome
B.Nursing interventions
C.Increased adherence to eye drop treatment
D.Patient with Glaucoma

72. Which of the following research evidence will MOST LIKELY provide you with the best available
evidence for adherence to eye drop treatment?
A.Systematic review of randomized controlled trials
B.Meta-synthesis
C.Meta-analysis
D.Experimental design
73. You would include as nursing interventions for adherence to eye drop treatment the following
EXCEPT:
A.Verbal/written information
B.Patient education program
C.Eye drop instillation training
D.Telephone instruction

74. Which of the following would be your measurement tools for adherence to eye drop treatment?
(SELECT ALL THAT APPLY):
1) Patient interview
2) Progress notes
3) Questionnaires
4) Checklist
5) Patient diary
A.1, 3, 5
B.1, 2, 4
C.2, 3, 4
D.3, 4, 5
75. For patients with impaired hand coordination, which of the following would you include in your
health teaching to ensure that there is adherence to eye drop treatment?
A.Install reminder devices
B.Teach patient’s caregiver how to instill eye drop
C.Simplify dosing regimen
D.Provide initial instruction

Situation
You are a staff nurse in the Ear Unit of a government hospital. As such, you are taking care of patients with different
ear conditions.
76. A patient came in with the chief complaints of ringing in the ears and a feeling of whirling around. The
attending physician suspects Meneire’s Syndrome. To arrive at a correct diagnosis, the patient must be
manifesting these three symptoms: paroxysmal whirling vertigo; tinnitus; and _____________-.
A. conductive hearing loss
B. bilateral symptoms
C. vomiting
D. sensorineural hearing loss
77.Which of the following is a predisposing factor for sensory hearing loss?
A. Serous otitis media
B. Ototoxic agents
C. Otosclerosis
D. Perforation of the eardrum
78. One of your patients underwent tympanoplasty. Which of the following interventions is the MOST
APPROPRIATE after a tympanoplasty?
A. Applying ear drops as ordered by the physician
B. Teaching the patient to avoid heavy physical activity for at least three weeks.
C. Rinsing ear three times a day using hydrogen peroxide as ordered
D. Avoiding the use of cotton in cleaning the ears.

79. Which of the following is useful in correcting a sensorineural hearing loss?


A. Nothing, the damage is irreversible
B. Myringotomy
C. Stapedectomy
D. Tympanolasty
80. Mel, a 21 year-old radio announcer came in for consult with the chief complaints of not being able to
hear well and inability to discriminate sounds he is hearing. An otoscopic examination was done to her,
which revealed that there is sensorineural loss. This condition is due to damage to what part of the ear?
A. Cochlea and auditory nerve
B. Vestibular nerve and cochlea
C. Middle ear and auditory nerve
D. Outer ear and cochlea

Situation
Research findings suggest that among soldiers who experienced war combats, there are some of them who may
experience the phenomenon known as Post-Traumatic Growth (PTG). You want to explore this phenomenon. You
formulated the problem as “What is the relationship between PTG and social support among soldiers who
experienced a battle-related traumatic amputation.

81. Which of the following is the independent variable of the study?


A.Social Support
B.PTG
C.Relationship
D.Soldiers with traumatic amputation
82. Which of the following is the dependent variable of the study?
A.Soldiers
B.Battle-related traumatic amputation
C.Social support
D.PTG
83. Which of the following sampling techniques is the MOST APPROPRIATE for this study?
A.Purposive sampling
B.Simple random sampling
C.Convenience sampling
D.Delphi technique
84. You have 1,656 soldiers who experienced battle-related traumatic amputation. Using Slovin’s formula
with a 5% margin of error, what will be your sample size.
A.996
B.490
C.540
D.1,200

85. Findings revealed an r=.49. This means the relationship between PTG and social support is a _________.
A.Low positive relationship
B.No relationship
C.Moderate positive relationship
D.High negative relationship

Situation
Ms. Alli, a 21 year old female student in a private university came in for consultations at the university Health
Services with the chief complaints of poor attention span, poor concentration, hearing some voices calling her and
sometimes seeing people following her. She claimed she started to experience all these three months ago.
86. Which of the following clusters of symptoms indicate the presence of Schizophrenia?
A.Unstable mood and delusions of grandeur
B.Hallucinations and delusions and decreased ability to function and society
C.Feelings of hopelessness and helplessness
D.Obsessive thoughts and ritualistic behavior
87. Which of the following marks the initial phase of schizophrenia?
A.Heightened work performance
B.Increased energy and motivation
C.Increased social interaction
D.Impaired role functioning and neglect of personal hygiene
88. Which of the following symptoms of Schizophrenia are likely to be responsive to medications?
A.Hallucinations
B.Social withdrawal
C.Apathy
D.Anhedonia

89. Which of the following methods can reduce the intensity of auditory hallucinations?
A.Giving the patient amitriptyline
B.Advising the patient not to pay attention to the voices
C.Encouraging the patient to read
D.Talking to the patients

90. What is the lifetime risk of developing schizophrenia in the general population?
A.15%
B.10%
C.1%
D.5%

Situation
In interdisciplinary care of a psychiatric patient: Each number of the health item has his/her roles and functions.
The following questions relate to this statement.

91. Which of the following is NOT a BARRIER to interdisciplinary collaboration?


A.Personal qualities of the individual health care team member that promote shared problem solving
B.Inequality of power and status
C.Inappropriate education and training of mental health team members
D.Role conflict

92. Which of the following is NOT an ingredient of the collaboration process?


A.Negotiations between each member of the health team to conceptualize new ways of solving problems
B.Communication techniques used by each health team member
C.Receptively and respect for each health team member’s contribution
D.Active and assertive contribution from each member of the health team
93. The element of collaboration essential in developing further psychiatric nursing is ________.
A.Link between nursing theory and clinical practice through research
B.Use of informatics
C.Collaborative practice
D.Utilization of evidence-based nursing interventions
94. The MOST important steps in interdisciplinary team building is _________.
A.Define disciplinary and individual roles and responsibilities
B.Use disciplinary treatment plan
C.Specify conflict resolution procedures
D.Communicate regularly, openly and clearly

95. The mental health team member is responsible for helping patients to receive the needed services is
the __________.
A.Case worker
B.Psychologist
C.Social worker
D.Psychiatric nurse
Situation.
The nurse’s work environment plays a vital role in her ability to provide quality patient care. Inadequate resources
in health care and unhealthy conditions in the workplace result to nurse’s work stress.
96. Burnout is a phenomenon that is BEST characterized by _____________.
A. emotional exhaustion: depersonalization: and reduced personal accomplishment
B. growing dissatisfaction among nurses due to changing career expectations.
C. increased absenteeism and fast turnover of nurses due to toxic work environment.
D. conflict between demands of work and family.

97.A common response to unhealthy work environment is physical tension. This can be overcome by
progressive muscle relaxation which basically involves __________.
A. focusing on an image to relax
B. active physical exercise like aerobics
C. releasing muscles from tension.
D. listening to a relaxation audio program.
98. Bio-behavioral interventions are being used increasingly by both nurses and patients in stress
management. The following are examples of such intervention except ___________.
A. progressive muscle relaxation
B. pharmacotherapy
C. guided imagery
D. mindful meditation

99. Allan, a young overweight nurse, is making five to ten sticks of cigarettes a day. He claimed that he
learned to smoke to relieve his stress caused by toxic assignments in the ward. Along with the smoking
he became a stress-eater. He decided to undergo a lifestyle modification which begins with ____________.
A. eating the right kind of foods
B. a recognition of the impact of unhealthy habits
C. doing a regular daily exercise.
D. a constructive and positive attitude in life.

100. Allan is a well-informed nurse on the smoking cessation program of the Department of Health. As a
next step to his decision to stop smoking, Allan needs ___________.
A. commitment
B. motivation
C. information
D. skills to implement change

You might also like